Logical Reasoning.pdf

  • Uploaded by: Balu Balaji
  • 0
  • 0
  • February 2021
  • PDF

This document was uploaded by user and they confirmed that they have the permission to share it. If you are author or own the copyright of this book, please report to us by using this DMCA report form. Report DMCA


Overview

Download & View Logical Reasoning.pdf as PDF for free.

More details

  • Words: 38,462
  • Pages: 224
Loading documents preview...
R

nithra

Logical Reasoning

Questions for Competitive Exams www.nithrabooks.com

[email protected]

Cell: 98659 24040

Copyright is reserved to the publisher, therefore the person who will try to imitate or try to print this book illegally or without the prior written permission of this publisher in any form, will be responsible for the loss and may be punished for compensation under copyright act.

Book/PDF Name : Logical Reasoning Questions for Competitive Exams Pages

: 224

Rate

: 100

Edition

: 2020

Copyright

: Publisher

In the compilation of this book all possible precautions have been taken to ensure that the informations provided is correct. Yet the publisher / authors will nto be held responsible for any printing errors or damage resulting from any inadvertent omission or inaccuracies in this book. However suggestions for the improvement of this book (Including printing errors, ommissions, etc. if any) are welcome and these will be incorporated in the subsequent editions of this book.

Published by :

P. Gokulanathan, Nithra Publications, AV Plaza 3rd & 4th Floor, South Car Street, Tiruchengode - 637211.

Logical Reasoning

Nithra

Logical Reasoning Analogy Solved Problems 1. Complete analogous pair: Automobile: Petrol::? A. fire : fuel B. plane : propeller C. diesel : gas D. man : food Answer: D. man: food Explanation: An automobile needs petrol to function. Similarly, man needs food in order to function. 2. Complete analogous pair: Calf: cow:: Puppy : ? A. cub B. kitten C. dog D. snake Answer: C. dog Explanation: A calf's parent is cow and puppy's parent is dog. 1

Logical Reasoning

Nithra

3. Complete analogous pair: Tungsten : Filament :: Tree : ? A. graphite B. paper C. wool D. fabric Answer: B. paper Explanation: The first term is used in the production of the second. 4. Complete analogous pair: Letter : Word :: A. homework : school B. club : people C. product : factory D. page : book Answer: D. page : book Explanation: A group of letters form a word. Similarly, a group of pages combined together to form a book.

2

Logical Reasoning

Nithra

5. Complete analogous pair. Brick : Clay :: Ruby : ? A. graphite B. corundum C. diamond D. pearl Answer: B. corundum Explanation: The second term is used in making the first. 6. Fill in the blanks with appropriate choices: Soap is to Dirt as Petrol is to …… A. Dry Cleaner B. Grease C. Car D. Clothes Answer: B. Grease Explanation: Soap is used to clean dirt based on similar pattern Petrol is used to clean Grease.

3

Logical Reasoning

Nithra

7.Happy is to Sad as Straight is to …… A. Line B. Rectangle C. Parallel D. Curve Answer: D. Curve Explanation: Happy is to Sad because they are antonym. Antonym of Straight is Circle from the given options. 8. Pleasure : Smile :: Pain : ? A. Aspirin B. Suffering C. Grimace D. Tranquillity Answer: C. Grimace Explanation: One smiles in pleasure while makes a grimace in pain.

4

Logical Reasoning

Nithra

9. Lawn : Grass :: ? A. Wool : sheep B. Skin : goat C. Rice : farm D. Pelt : fur Answer: D. Pelt : fur Explanation: As grass grows on lawn similarly, fur grows on pelt. Second grows on the first. 10. Dwan : Twilight :: ? A. Prologue : epilogue B. Day : evening C. Night : day D. Minute : hour Answer: A. Prologue : epilogue Explanation: Dawn is beginning of day while twilight is end of day. Similarly, a prologue come at the beginning of the book & epilogue at the end of book.

5

Logical Reasoning

Nithra

Questions for Practice 1. Moon : Satellite : : Earth : ? (a) Sun

(b) Planet

(c) Solar System

(d) Asteroid

2. Forecast : Future : : Regret : ? (a) Present

(b) Atone

(c) Past

(d) Sins

3. Influenza : Virus : : Typhoid: ? (a) Bacillus

(b) Parasite

(c) Protozoa

(d) Bacteria

4. Fear : Threat 2 : Anger : ? (a) Compulsion

(b) Panic

(c) Provocation

(d) Force

5. Melt : Liquid 2 : Freeze: ? (a) Ice

(b) Condense

(c) Solid

(d) Crystal

6

Logical Reasoning

Nithra

Artificial Language Solved Problems 1. Here are some of the words translated from an artificial language. If creektulo means treeplantation; creekfuos means treemutation; fuosseed means mutationsapling; Which word means sapling? A. Creek B. Tulo C. Fuos D. Seed Answer: D. Seed Explanation: ✓ tree means creek; ✓ mutation means fuos; ✓ sapling means seed. 2. Here are some words translated from an artificial language. jalkamofti means happy birthday; moftihoze means birthday party; mentogunn means goodness; Which word could mean ''happiness''? A. jalkagunn B. mentohoze C. moftihoze D. hozemento Answer: A. jalkagunn 7

Logical Reasoning

Nithra

Explanation: ✓ ✓ ✓ ✓ ✓ ✓

Jalka means happy; mofti means birthday; hoze means party; mento means good; and gunn means ness; Thus, jalkagunn means happiness.

3. Here are some of the words translated from an artificial language. If robbhood means nextgeneration; hoodgough means generationproud; goughspur means proudnation; Which word means generation? A. robb B. hood C. gough D. spur Answer: B. hood Explanation: By comparing first and second sentences, we can deduce the meaning of generation.

8

Logical Reasoning

Nithra

4. Here are some words translated from an artificial language. lelibroon means yellowhat; plekafroti means flowergarden; frotimix means gardensalad; Which word could mean ''yellowflower''? A. lelifroti B. lelipleka C. plekabroon D. frotibroon Answer: B. lelipleka Explanation: yellow means leli; flower means pleka. 5. Here are some words translated from an artificial language. granamelke means big tree; pinimelke means little tree; melkehoon means tree house; Which word could mean ''big house''? A. granahoon B. pinishur C. pinihoon D. melkegrana Answer: A. granahoon

9

Logical Reasoning

Nithra

Explanation: ✓ ✓ ✓ ✓ ✓

Grana means big; melkemeans tree; pini means little; hoonmeans house. Therefore, granahoon means big house.

6. The question that follow will ask you to reverse the process and translate an English word into the artificial language. Here are some words translated from an artificial language. malgauper means peach cobbler malgaport means peach juice moggagrop means apple jelly Which word could mean "apple juice"? A. malgaauper B. moggaport C. gropport D. moggagrop Answer: B. moggaport Explanation: ✓ ✓ ✓ ✓ ✓ ✓

Malga means peach; uper means cobbler; port means juice; mogga means apple; and grop means jelly. Therefore, moggaport means apple juice.

10

Logical Reasoning

Nithra

7. The question that follow will ask you to reverse the process and translate an English word into the artificial language. Here are some words translated from an artificial language. moolokarn means blue sky wilkospadi means bicycle race moolowilko means blue bicycle Which word could mean "race car"? A. wilkozwet B. spadiwilko C. moolobreil D. spadivolo Answer: D. spadivolo Explanation: From wilkospadi, you can determine that wilko means bicicyle and spadi means race. Therefore, the first part of the word that means racecar should begin with spadi. That limits your choices to b and d. Choice b, spadiwilko, is incorrect because we have already determined that wilko means bicycle. Therefore, the answer must be choice d, spadivolo.

8. The question that follow will ask you to reverse the process and translate an English word into the artificial language. Here are some words translated from an artificial language. morpirquat means birdhouse beelmorpir means bluebird beelclak means bluebell Which word could mean "houseguest"? A. clakquat B. quathunde C. beelmoki 11

Logical Reasoning

Nithra

D. morpirhunde Answer: B. quathunde Explanation: ✓ ✓ ✓ ✓ ✓

Morpir means bird; quat means house; beel means blue; clak means bell. Choice B, which begins with quat, is the only possible option.

9. The question that follow will ask you to reverse the process and translate an English word into the artificial language. Here are some words translated from an artificial language. relftaga means carefree otaga means careful fertaga means careless Which word could mean "aftercare"? A. relffer B. tagazen C. tagafer D. zentaga Answer : B. tagazen Explanation : In this language, the root word taga, which means care, follows the affix (relf, o-, or fer-). Therefore, in the word aftercare, the root word and the affix would be reversed in the artificial language. The only choice, is tagazen, because tagafer would mean less care.

12

Logical Reasoning

Nithra

10. The question that follow will ask you to reverse the process and translate an English word into the artificial language. Here are some words translated from an artificial language. slar means jump slary means jumping slarend means jumped Which word could mean "playing"? A. ellaclarg B. slarmont C. clargy D. clargslarend Answer : C. clargy Explanation : According to this language, slar means jump. The suffix "ing" is represented by "y". Since choice C is the only one that ends in the letter y, this is the only possible option.

Questions for Practice 1. Here are some words translated from an artificial language. gorblflur means fan belt pixngorbl means ceiling fan arthtusl means tile roof Which word could mean "ceiling tile"? A. B. C. D.

gorbltusl flurgorbl arthflur pixnarth

13

Logical Reasoning

Nithra

2. Here are some words translated from an artificial language. hapllesh means cloudburst srenchoch means pinball resbosrench means ninepin Which word could mean "cloud nine"? A. B. C. D.

leshsrench ochhapl haploch haplresbo

3. Here are some words translated from an artificial language. agnoscrenia means poisonous spider delanocrenia means poisonous snake agnosdeery means brown spider Which word could mean "black widow spider"? A. B. C. D.

deeryclostagnos agnosdelano agnosvitriblunin trymuttiagnos

4. Here are some words translated from an artificial language. moolokarn means blue sky wilkospadi means bicycle race moolowilko means blue bicycle Which word could mean "racecar"? A. B. C. D.

wilkozwet spadiwilko moolobreil spadivolo

14

Logical Reasoning

Nithra

5. Here are some words translated from an artificial language. migenlasan means cupboard lasanpoen means boardwalk cuopdansa means pullman Which word could mean "walkway"? A. B. C. D.

poenmigen cuopeisel lasandansa poenforc

Assertion and Reason Solved Problems In each of the following questions, there is a Assertion and two Reasons, choose the correct alternative from the following: A. Both A and R are true and R is the correct explanation of A. B. Both A and R are true but R is NOT the correct explanation of A. C. A is true but R is false. D. A is false but R is true. 1. Assertion (A): Bangladesh imports jute from India. Reason (R): Bangladesh has most of the jute mills. Answer: D. A is false but R is true. Explanation: When Bangladesh was created after partition of India, the areas of jute production went to Bangladesh while the jute mills were left in India. So, India imports raw jute from Bangladesh.

15

Logical Reasoning

Nithra

2. Assertion (A): India is facing the problem of inflation. Reason (R) : We have failed to check the growth of black money. Answer: A. Both A and R are true and R is the correct explanation of A. Explanation: Inflation in India is caused by unrestricted growth of black money. 3. Assertion (A): Leaves of plants are green. Reason (R) : Plants contain chromoplasts, the green pigment. Answer: C . A is true but R is false. Explanation: Leaves of plants are green because they contain the green pigment, chlorophyll. However, plants contain chromoplasts but they are not green pigments. 4. Assertion (A): Bronze is used for making statues. Reason (R) : Bronze is an alloy of copper and trin. Answer: B. Both A and R are true but R is NOT the correct explanation of A. Explanation: Bronze is an alloy of copper and tin. It is resistant to corrosion and so it is used to make statues.

16

Logical Reasoning

Nithra

5. Assertion (A): In India, the judiciary is independent of the executive. Reason (R) : Judiciary favours the government and helps in the implementation of its plans. Answer: C . A is true but R is false. Explanation: In India, the judiciary is completely independent of the executive. It has no interference in the affairs of the state nor can it be influenced by the government. 6. Assertion (A): Increase in carbon dioxide would melt polar ice. Reason (R): Global temperature would rise. Answer: A. Both A and R are true and R is the correct explanation of A. Explanation: ✓ ✓ ✓ ✓

The carbon dioxide envelope in earth's atmosphere traps the heat. With increase in the proportion of carbon dioxide, therefore, the global temperature would rise, thus causing the polar ice to melt.

7.Assertion (A): Tamil Nadu gets most of the rainfall in winter. Reason (R) : Tamil Nadu gets rainfall from retreating monsoons. Answer: A . Both A and R are true and R is the correct explanation of A. Explanation: Rainfall in Tamil Nadu is caused by the retreating monsoons which occur in winter.

17

Logical Reasoning

Nithra

8. Assertion (A) : In India, females have higher life expectancy than the males. Reason (R) : Females receive a better diet. A. Both A and R are true and R is the correct explanation of A. B. Both A and R are false. C. A is true but R is false. D. A is false but R is true. E. Both A and R are true but R is NOT the correct explanation of A. Answer: E. Both A and R are true but R is NOT the correct explanation of A. Explanation : In India, due to high birth rate and due to neglect, females have a lower lift expectancy than the males and although females need a better diet, they do not receive it. 9. Assertion (A) : Telephone wires sag more in summer. Reason (R) : They expand due to summer heat. A. Both A and R are true but R is NOT the correct explanation of A. B. Both A and R are true and R is the correct explanation of A. C. A is false but R is true. D. A is true but R is false. E. Both A and R are false. Answer: B. Both A and R are true and R is the correct explanation of A. Explanation : The metal of telephone wires expands in summer and the wires become loose. So, they say.

18

Logical Reasoning

Nithra

10. Assertion (A) : India is a democratic country. Reason (R) : India has a Constitution of its own. A. Both A and R are true and R is the correct explanation of A. B. Both A and R are true but R is NOT the correct explanation of A. C. A is true but R is false. D. A is false but R is true. E. Both A and R are false. Answer : B. Both A and R are true but R is NOT the correct explanation of A. Explanation : India is a democratic country because its government is the government of the people, for the people and by the people. It is also true that India has its own Constitution.

Questions for Practice In each of the questions given below, there are two statements marked as Assertion (A) and Reason (R). Mark your answer as per the codes provided below: A. Both A and R are true and R is the correct explanation of A. B. Both A and R are true but R is not the correct explanation of A. C. A is true but R is false. D. A is false but R is true. E. Both A and R are false. 1. Assertion (A): When lightning strikes, the sound is heard a little after the flash is seen. Reason (R): The velocity of light is greater than that of the sound. 2. Assertion (A): A ship rises as it enters the sea from a river. Reason (R): The density of sea water is higher as compared to river water.

19

Logical Reasoning

Nithra

3. Assertion (A): Indian President is the head of the State. Reason (R): Indian Parliament consists of the President, Lok Sabha and Rajya Sabha. 4. Assertion (A): The British sovereignty continued to exist in f India. Reason (R): The British sovereign appointed the last Governor-General of free India. 5. Assertion (A): Sprouting should not be done before consuming the grains. Reason (R): Sprouting kills many vital vitamins.

Coding and Decoding Definition Coding is a process used to encrypt a word, a number in a particular code or pattern based on some set of rules. Decoding is a process to decrypt the pattern into its original form from the given codes. Letter Coding In this type of questions, alphabets of a word are replaced by some other alphabets according to specific rule to form code. Number Coding In this type of questions, a word is replaced by certain numbers according to some specific rule.

20

Logical Reasoning

Nithra

Solved Problems Study the following information arrangement carefully and answer the questions given below: With a certain code language, The meeting adjourned abruptly is written as !e48 @h08 #b63 $d80 Since India gained Internet is written as #n63 -a35 %i24 %n24 Scientists have discovered bacteria is written as *a15 #a63 &c99 &i99 Current economic scenario fine is written as #c63 *i15 #c63 !u48 1. What is the code for abruptly? A. %i24 B. #b63 C. -a35 D. %n24 Answer : B. #b63 Explanation : ✓ ✓ ✓ ✓ ✓ ✓ ✓ ✓ ✓ ✓ ✓

The pattern for this coding is Each symbol is denoted by number of words like For third letter word is @ For fourth letter word is * For fifth letter word is % For sixth letter word is For seventh letter word is ! For eight letter words is # For nineth letter word is $ For tenth letter word is & So, abruptly is eight letter word. For eigth letter word is # The correct option is B.

21

Logical Reasoning

Nithra

2. What is the code for Since meeting discovered? A. *a15 !e48 *i99 B. %i24 @h48 *i99 C. %i24 !e48 *#c63 D. %i24 !e48 &i99 Answer : D. %i24 !e48 &i99 Explanation : ✓ ✓ ✓ ✓ ✓ ✓ ✓

For fifth letter word is % For seventh letter word is ! For tenth letter word is & Each alphabet is a second letter of the each word like For since - i For meeting - e For discovered - i The correct option is D.

3. What does &c99 @i99 #n63 @h08 stand for? A. sine India have fine B. The scientists discovered internet C. meeting gained the economic D. current meeting have internet Answer : B. The scientists discovered internet Explanation : ✓ ✓ ✓ ✓ ✓

Each number is stands for total number of words square - 1 For third letter word = 32 - 1 = 9 - 1 = 8 For fourth letter word = 42 - 1 = 16 - 1 = 15 For fifth letter word = 52 - 1 = 25 - 1 = 24 For sixth letter word = 62 - 1 = 36 - 1 = 35 22

Logical Reasoning ✓ ✓ ✓ ✓ ✓

Nithra

For seventh letter word = 72 - 1 = 49 - 1 = 48 For eight letter word = 82 - 1 = 64 - 1 = 63 For nineth letter word = 92 - 1 = 81 - 1 = 80 For tenth letter word = 102 - 1 = 100 - 1 = 99. So, 8= The; 99 - Scientists; 63 – internet.

4. What could be the code for economic internet? A. #n63 #c63 B. %i24 *a15 C. !e48 #n63 D. #c63 -a35 Answer : A. #n63 #c63 Explanation : ✓ ✓ ✓ ✓ ✓

Economic is eight letter word. So For eight letter word = 82 - 1 = 64 - 1 = 63 Economic = 63 Internet is eight letter word. So the same reason. Internet = 63

5. In a certain code, BOXER is written as AQWGQ. How VISIT is written in that code? A. UKRKU B. UKRKS C. WKRKU D. WKRKS Answer : B. UKRKS

23

Logical Reasoning

Nithra

Explanation : Letters at the odd places have been written one letter back, and letters at the even places have been written two letters ahead in the coded word as their positions in the alphabet. 6. If PAPER is written as OZODQ, how PENCIL can be written in that code? A. QFODJM B. OFOBHM C. ODMDJM D. ODMBHK Answer : D.ODMBHK Explanation : Each letter has been coded one letter back in the alphabetical series. 7. If in a code language REMOTE is coded as ROTEME, which word would be coded as PNIICC? A. PNIICC B. PICCIN C. PINCIC D. PICNIC Answer : D. PICNIC Explanation : The word has been coded by simply interchanging the position of the letters.

24

Logical Reasoning

Nithra

8. In a certain code, bright fresh sunny day means cin bin zin hin; scent of fresh flower means din zin lin bin; bright light of trucks; means lin min hin rin; trucks loaded with flowers means fin nin din min. What is the code for bright in that language? A. bin B. lin C. hin D. zin Answer : C. Hin Explanation : In the first and third sentences, common word is bright and the common code is hin. 9. Study the following information to answer the questions that follow. In a certain code il be pee means roses are blue, sik hee means red flowers and pee nut hee means flowers are vegetables. How is red written in that code? A. hee B. sik C. be D. Cannot be determined Answer : B. Sik Explanation : From the second and third sentences, it is clear that hee stands for flowers. Hence, from second sentence red stands for sik.

25

Logical Reasoning

Nithra

10. If in a certain language, MADRAS is coded as NBESBT, how is BOMBAY coded in that code? A.CPNCBX B.CPNCBZ C.CPOCBZ D.CQOCBZ E. None of these Answer : B. CPNCBZ Explanation : Each letter in the word is moved one step forward to obtain the corresponding letter of the code.

Questions for Practice 1. In a certain code 'LIMCA' is written as 'HJLDZ'. Which of the following words is written as 'IFWJBP'? a) M E X I C O

b) MERCURY

c) JAPAN

d) MIDNIGHT

e) H O N D U S

2. If in the English alphabet, every alternate letter from B onwards is written in small letters while others are written in capitals, then how will the 3rd day from Tuesday will be coded? a) W e D N e S d A Y AY e) f r I d A Y

b) W E d n E S d A Y

26

c) T H U R S d A Y

d) T h U r S d

Logical Reasoning

Nithra

3. In a certain code 'SEQUENCE' is coded as 'FDOFVRFT. How is 'CHILDREN' coded in that code? a) OFESJMID these

b) OFSEMJID

c) OFSEJIMD

d) OFSEJMID

e) None of

4. If the letters of the word 'CYCLINDER' are arranged alphabetically, then which letter would be farthest from the first letter of word? a) N

b) E

c) Y

d) R

e) None of these

5. In certain code 'FROZEN' is written as 'OFAPSG'. Then how would 'MOLTEN' be written in that code? a) OFPOMN

b) OFSMPN

c) OFUMPN

d) OFUNPM

e) OFUMON

Course of Action Solved Problems 1. Statement: Use Kajaria Ceramics tiles to decorate your house- An advertisement. Courses of Action: I. People want to decorate their house. II.Only ceramic tiles can make a house decorative. A. If only assumption I is implicit. B. Only II follows If only assumption II is implicit. C. If either I or II is implicit. D. If both I and II are implicit. E. If neither I or II is implicit. Answer : A. If only assumption I is implicit.

27

Logical Reasoning

Nithra

Explanation: Obviously, assumption I is implicit in the statement. The fact which is highlighted or mentioned in the advertisement is liked or looked for by the people. The use of word "only" makes the second assumption invalid. 2. Statement: The Officer Incharge of a company had a hunch that some money was missing from the safe. Courses of Action: I. He should get it recounted with the help of the staff and check it with the balance sheet. II. He should inform the police. A. If Only I follows B. If Only II follows C. If Either I or II follows D. If Neither I nor II follows E. If Both I and II follow Answer : A. If Only I follows Explanation : Clearly, a suspicion first needs to be confirmed and only when it is confirmed, should an action to be taken. So, only course I follows

28

Logical Reasoning

Nithra

3. Statement: It is futile to discuss corruption at any forum. Corruption is rampant everywhere in the country. Courses of Action: I. Corruption is inevitable in any system. II. Corruption is a serious concern. A. If only assumption I is implicit. B. Only II follows If only assumption II is implicit. C. If either I or II is implicit. D. If both I and II are implicit. E. If neither I or II is implicit. Answer : E. If neither I or II is implicit. Explanation : Both the assumptions are more or less personal views. Therefore, neither of these assumptions is implicit in the statement. 4. Statement: Since its launching in 1881, Vayudoot has so far accumulated losses amounting to Rs. 153 crore. Courses of Action: I. Vayudoot should be directed to reduce wasteful expenditure and to increase passenger fare. II. An amount of about Rs 300 crore should be provided to Vayudoot to make the airliner economically viable. Answer: Only I follows Explanation: Clearly, for better economic gain, losses should be reduced and income increased. So, only course I follows. 29

Logical Reasoning

Nithra

5. Statement: A large number of people in ward X of the city are diagnosed to be suffering from a fatal malaria type. Courses of Action: I. The city municipal authority should take immediate steps to carry out extensive fumigation in ward X. II. The people in the area should be advised to take steps to avoid mosquito bites. Answer: Both I and II follow Explanation: Clearly, prevention from mosquitoes and elimination of mosquitoes are two ways to prevent malaria. So, both the courses follow. 6.Statement: Exporters in the capital are alleging that commercial banks are violating a Reserve Bank of India directive to operate a post shipment export credit denominated in foreign currency at international rates from January this year. Courses of Action: I. The officers concerned in the commercial banks are to be suspended. II. The RBI should be asked to stop giving such directives to commercial banks. Answer: Neither I nor II follows Explanation: The statement mentions that the commercial banks violate a directive issued by the RBI. The remedy is only to make the banks implement the Act. So, none of the courses follows.

30

Logical Reasoning

Nithra

7. Statement: There is a considerable increase in the number of persons affected by water-borne diseases during monsoon period. Courses of Action: I. The question should be raised in the Legislative Assembly. II. The Government should disseminate adequate information regarding the pure drinking water to people. III. All the hospitals in the city should be equipped properly for the treatment of patients during monsoon period. Answer: Only II and III follow Explanation: Any aspect of health has two factors to tackle with - prevention and treatment. Prevention includes creating awareness among people, and treatment includes providing adequate medical facilities. So, both II and III follow. 8. Statement: Some serious blunders were detected in the Accounts section of a factory. Courses of Action: I. An efficient team of auditors should be appointed to check the Accounts. II. A show cause notice should be issued to all the employees involved in the irregularity. Answer: Both I and II follow Explanation: Clearly, the situation demands that the faults in Accounts be properly worked out and the persons involved be interrogated about the matter. So, both the courses follow.

31

Logical Reasoning

Nithra

9. Statement: Duty free technology parks where foreign firms can manufacture electronic hardware components are proposed to be established at various places in the country. Courses of Action: I. Government should immediately implement the proposal to augment the foreign currency reserve by exporting the products. II. Government should not implement the proposal as it will hinder indigenous production of hardware components. A. Only I follows B. Only II follows C. Either I or II follows D. Neither I nor II follows E. Both I and II follow Answer : A. Only I follows Explanation : The step discussed in the statement would surely give a boost to hardware industry and help our country to stand apart in this field. Thus, only I follows. 10. Statement: The Finance Minister submits his resignation a month before the new budget is to be presented in the Parliament. Courses of Action: I. The resignation should be accepted and another person should be appointed as the Finance Minister. II. The resignation should not be accepted. A. Only I follows B. Only II follows 32

Logical Reasoning

Nithra

C. Either I or II follows D. Neither I nor II follows E. Both I and II follow Answer : B. Only II follows Explanation : Clearly, an already working Finance Minister shall know better all the plans and resources of the Government and he alone can present a suitable budget. So, course II follows.

Questions for Practice 1. Statement: A large number of people in ward X of the city are diagnosed to be suffering from a fatal malaria type. Courses of Action: The city municipal authority should take immediate steps to carry out extensive fumigation in ward X. The people in the area should be advised to take steps to avoid mosquito bites. A. Only I follows B. Only II follows C. Either I or II follows D. Neither I nor II follows E. Both I and II follow

33

Logical Reasoning

Nithra

2. Statement: Severe drought is reported to have set in several parts of the country. Courses of Action: Government should immediately make arrangement for providing financial assistance to those affected. Food, water and fodder should immediately be sent to all these areas to save the people and cattle. A. B. C. D. E.

Only I follows Only II follows Either I or II follows Neither I nor II follows Both I and II follow

3. Statement: Since its launching in 1981, Vayudoot has so far accumulated losses amounting to Rs 153 crore. Courses of Action: Vayudoot should be directed to reduce wasteful expenditure and to increase passenger fare. An amount of about Rs 300 crore should be provided to Vayudoot to make the airliner economically viable. A. B. C. D. E.

Only I follows Only II follows Either I or II follows Neither I nor II follows Both I and II follow

34

Logical Reasoning

Nithra

4. Statement: Exporters in the capital are alleging that commercial banks are violating a Reserve Bank of India directive to operate a post shipment export credit denominated in foreign currency at international rates from January this year. Courses of Action: The officers concerned in the commercial banks are to be suspended. The RBI should be asked to stop giving such directives to commercial banks. A. B. C. D. E.

Only I follows Only II follows Either I or II follows Neither I nor II follows Both I and II follow

5. Statement: A large number of people die every year due to drinking polluted water during the summer. Courses of Action: The government should make adequate arrangements to provide safe drinking water to all its citizens. The people should be educated about the dangers of drinking polluted water. A. B. C. D. E.

Only I follows Only II follows Either I or II follows Neither I nor II follows Both I and II follow

35

Logical Reasoning

Nithra

Cube and Dice Definition "Cube" means to cut food into pieces that are even, like a square. The size is usually about the same as the chopped pieces sizes; about 1/3 to 1/2". "Dice" means to cut food into even, small squares about 1/4" in diameter. And "mince" means to cut foods into even, very small pieces about 1/8" in diameter.

Solved Problems 1. If total number of cuts is 10 then find the minimum number of pieces that can be obtained. Answer : 11 Explanation : If total number of cuts is 10 then minimum number of pieces is 11. When cut is made in one plane only. 2. If total number of cuts is 10 then find the maximum number of pieces that can be obtained. Answer : 80 Explanation : If total number of cut is 10 then for maximum number of pieces these cuts have to be well distributed in three planes. For 10 cuts, 3,3 and 4 is the distribution of cuts. Hence total number of pieces is (3 + 3)(3 + 1)(4 + 1) = 4 * 4 * 5 = 80 36

Logical Reasoning

Nithra

3. How many of the cubes have at most faces painted? Answer : 208 Explanation : Since total number of cubes is hence in the formula, we will substitute n = 6 At most 2 faces painted = number of cubes with 0 face painted + number of cubes with 1 face painted + number of cubes with 2 face painted = 64 + 48 + 96 = 208 4. If total number of pieces (Smaller cubes/cuboids) is 45 then find the possible number of cuts. Answer : 8 or 12 Explanation : If 45 = 1 * 1 * 45 then we require only 44 cuts in one plane. If 1 * 3 * 15 then we require 2 cuts in one plane and 14 cuts in other plane so total number of cuts is 2 + 14 = 16. If 1 * 5 * 9 the we require 4 cuts in one plane and 8 cuts in other plane so total number of cuts is 4 + 8 = 12 If 3 * 3 * 5 then we require 2 cuts in one plane, 2 cuts in 2nd plane and 4 cuts in 3rd plane so total number of cuts is 2 + 2 + 4 = 8.

37

Logical Reasoning

Nithra

5. If total number of cuts is 20 then find the ratio of maximum and minimum of pieces that can be obtained. Answer : 448:21 Explanation : For maximum number of pieces cuts has to be 6, 7 and 7. Maximum number of pieces is (6 + 1)(7 + 1)(7 + 1) = 7 * 8 * 8 = 448. Minimum number of pieces is 20 + 1 = 21. Hence required ratio is 448:21 6. How many of the cubes have at least 2 faces painted? Answer : 104 Explanation : Since total number of cubes is hence in the formula we will substitute n = 6 At least 2 faces painted = number of cubes with 2 face painted + number of cubes with 3 face painted = 96 + 8 = 104. 7. 216 cubes of similar size are arranged in the form of the bigger cube (6 cubes on each side, i.e..., 6 * 6 * 6) all the exposed surfaces are painted. How many of the cubes have 0 faces painted? Answer : 64 Explanation : Since total number of cubes is hence in the formula we will substitute n = 6 Number of the cubes with 0 faces painted = (6 - 2)3 = 43 = 64

38

Logical Reasoning

Nithra

8. A dice is numbered from 1 to 6 in different ways. If 1 is adjacent to 2, 4 and 6, then which of the following statements is necessarily true? A) 2 is opposite to 6 B) 1 is adjacent to 3 C) 3 is adjacent to 5 D) 3 is opposite to 5 Answer : C) 3 is adjacent to 5 Explanation : If 1 is adjacent to 2, 4 and 6 then either 3 or 5 lies opposite to 1. So, the numbers 3 and 5 cannot lie opposite to each other. Hence, 3 is adjacent to 5 (necessarily). 9. Four usual dice are thrown on the ground. The total of numbers on the top faces of these four dice is 13 as the top faces showed 4, 3, 1 and 5 respectively. What is the total of the faces touching the ground? A) 12 B) 13 C) 15 D) Cannot be determined Answer : C) 15 Explanation : In a usual dice, the sum of the numbers on any two opposite faces is always 7. Thus, 1 is opposite 6, 2 is opposite 5 and 3 is opposite 4. Consequently, when 4, 3, 1 and 5 are the numbers on the top faces, then 3, 4, 6 and 2 respectively are the numbers on the face touching the ground. The total of these numbers = 3 + 4 + 6 + 2 = 15.

39

Logical Reasoning

Nithra

10. A dice is numbered from 1 to 6 in different ways. If 1 is adjacent to 2, 3 and 5, then which of the following statements is necessarily true? A) 4 is adjacent to 6 B) 2 is adjacent to 5 C) 1 is adjacent to 6 D) 1 is adjacent to 4 Answer : A) 4 is adjacent to 6 Explanation : If 1 is adjacent to 2, 3 and 5, then either 4 or 6 lies opposite to 1. So, the numbers 4 and 6 cannot lie opposite to each other. Hence, 4 necessarily lies adjacent to 6.

Questions for practice 1. Find the number of cubes with two surfaces painted with red and black color. A. 10

B. 8 0

C. 9

D. 16

2. Four usual dice are thrown on the ground. The total of numbers on the top faces of these four dice is 13 as the top faces showed 4, 3, 1 and 5 respectively. What is the total of the faces touching the ground? A. 12

B. 13

C. 15

D. Cannot be determined

3. A dice is numbered from 1 to 6 in different ways. If 1 is adjacent to 2, 4 and 6, then which of the following statements is necessarily true? A) 2 is opposite to 6 B) 1 is adjacent to 3

C) 3 is adjacent to 5 D) 3 is opposite to 5

40

Logical Reasoning

Nithra

4. A cube is painted blue on two adjacent surfaces and black on the surfaces opposite to blue surfaces and green on the remaining faces .Now the cube is cut into 216 smaller cubes of equal size. A) 56

B) 48

C) 32 D) 64

5. A dice is numbered from 1 to 6 in different ways. If 1 is opposite to 5 and 2 is opposite to 3, then A) 4 is adjacent to 3 and 6 C) 4 is adjacent to 5 and 6

B) 2 is adjacent to 4 and 6 D) 6 is adjacent to 3 and 4

Essential Part Solved Problems 1. Choose the word that is necessary for the word given in the question: Saddle A. Horse B. Seat C. Stirrups D. Horn Answer : B. Seat Explanation : A saddle is something one uses to sit on an animal, so it must have a seat (choice b). A saddle is often used on a horse (choice a), but it may be used on other animals. Stirrups (choice c) are often found on a saddle but may not be used. A horn (choice d) is found on Western saddles, but not English saddles, so it is not the essential element here.

41

Logical Reasoning

Nithra

2. Choose the word that is necessary for the word given in the question: Cell A. Running B. Swimming C. Winning D. Speaking Answer : C. Winning Explanation : Without a first-place win, there is no champion, so winning is essential. There may be champions in running, swimming, or speaking, but there are also champions in many other areas. 3. Choose the word that is necessary for the word given in the question: Glacier A. Mountain B. Winter C. Prehistory D. Ice Answer : D. Ice Explanation : A glacier is a large mass of ice and cannot exist without it. A glacier can move down a mountain, but it can also move across a valley or a plain, which rules out choice a. Glaciers exist in all seasons, which rules out choice b. There are many glaciers in the world today, which rules out choice c.

42

Logical Reasoning

Nithra

4. Choose the word that is necessary for the word given in the question: Antique A. Rarity B. Artifact C. Aged D. Prehistoric Answer : C. Aged Explanation : An antique is something that belongs to, or was made in, an earlier period. It may or may not be a rarity (choice a), and it does not have to be an artifact, an object produced or shaped by human craft (choice b). An antique is old but does not have to be prehistoric (choice d). 5. Choose the word that is necessary for the word given in the question: Orchestra A. Violin B. Stage C. Musician D. Soloist Answer : C. Musician Explanation : An orchestra is a large group of musicians, so musicians are essential. Although many orchestras have violin sections, violins aren't essential to an orchestra (choice a). Neither a stage (choice b) nor a soloist (choice d) is necessary.

43

Logical Reasoning

Nithra

6. Choose the word that is necessary for the word given in the question: Infirmary A. Patient B. Disease C. Surgery D. Receptionist Answer : A. Patient Explanation : An infirmary is a place that takes care of the infirm, sick, or injured. Without patients, there is no infirmary. Surgery (choice C) may not be required for patients. A disease (choice B) is not necessary because the infirmary may only see patients with injuries. A receptionist (choice D) would be helpful but not essential. 7. Choose the word that is necessary for the word given in the question: Guitar A. Band B. Strings C. Songs D. Teacher Answer : B. Strings Explanation : A guitar does not exist without strings, so strings are an essential part of a guitar. A band is not necessary to a guitar (choice A). Guitar playing can be learned without a teacher (choice D). Songs are by products of a guitar (choice C).

44

Logical Reasoning

Nithra

8. Choose the word that is necessary for the word given in the question: Domicile A. Tenant B. House C. Kitchen D. Dwelling Answer : D. Dwelling Explanation : A domicile is a legal residence, so dwelling is the essential component for this item. You do not need a tenant (choice A) in the domicile, nor do you need a kitchen (choice C). A house (choice B) is just one form of a domicile (which could also be a tent, Hogan, van, camper, motor home, apartment, dormitory, etc.). 9. Choose the word that is necessary for the word given in the question: Vertebrate A. Mammal B. Reptile C. Backbone D. Animal Answer : C. Backbone Explanation : All vertebrates have a backbone. Reptiles (choice B) are vertebrates, but so are many other animals. Mammals (choice A) are vertebrates, but so are birds and reptiles. All vertebrates (choice D) are animals, but not all animals are vertebrates.

45

Logical Reasoning

Nithra

10. Choose the word that is necessary for the word given in the question: Hurricane A. Beach B. Wind C. Damage D. Cyclone Answer : B. Wind Explanation : A hurricane cannot exist without wind. A beach is not essential to a hurricane (choice A). A hurricane is a type of cyclone, which rules out (choice D). Not all hurricanes cause damage (choice C).

Questions for Practice 1. Choose the word that is necessary for the word given in the question: harvest A. autumn B. stockpile C. tractor D. crop 2. Choose the word that is necessary for the word given in the question: desert A. cactus B. arid C. oasis D. flat

46

Logical Reasoning

Nithra

3. Choose the word that is necessary for the word given in the question: book A. fiction B. pages C. pictures D. learning 4. Choose the word that is necessary for the word given in the question: language A. tongue B. slang C. writing D. words 5. Choose the word that is necessary for the word given in the question: school A. student B. report card C. test D. learning

Inferences Solved Problems 1. Should the teachers be stopped from beating the students? Arguments: I. Yes, child psychologies say that beating hinders the learning process in a child. II. No, spare the cane and spoil the child. A. If either I or II is strong. B. If only argument II is strong. C. If only argument I is strong. D. If both I and II are strong. 47

Logical Reasoning

Nithra

Answer : C. If only argument I is strong. Explanation : Anything which hinders the learning process in a child must be controlled and, if possible, be prohibited. The aim of teacher should be to make a child learn. According to argument I, beating causes hindrance to learning process of a child thus defeating the ultimate goal. If also supports its argument by referring to the findings of psychologists. Hence, I is a strong argument. Argument II states that a child will be spoiled if it is not caned. Caning is an extreme measure, there are also other softer methods to rectify a child. Moreover, the argument is not substantiated by any facts. Hence, II is a weak argument. 2. Should the director with political influence be preferred to the director with vast experience for the post of managing director? Arguments: I. Yes, now a days, business becomes successful by exercising influence rather than using strategies. II. No, strategies developed by an experienced businessman makes a business definitely successful. A. If only argument I is strong. B. If neither I nor II is strong. C. If only argument II is strong. D. If either I or II is strong. Answer : C. If only argument II is strong. Explanation : "Influence" may be interpreted in more than one way. Statement I says that

48

Logical Reasoning

Nithra

"influences" is better than "strategies" but it is not referring directly to "political influence" which is what is given in the passage. Hence, this is a weak argument. "Experience" is to be constructed in only one way, that is experience in developing strategies which is a part of the job of a director. Hence this is a strong argument. 3. Should rail-roads be privatised? Arguments: I. Yes, it is not possible to supervise an organization as big as a rail-road by any private company or person. II. No, privatisation makes train-journey expensive. A. if both I and II are strong. B. if only argument II is strong. C. if either I or II is strong. D. if neither I nor II is strong. Answer : A. if both I and II are strong. Explanation : We cannot check the validity of the two given statements but if both are true, each will present a strong argument.

49

Logical Reasoning

Nithra

4. Should the institute conduct classes in remote villages? Arguments: I. Yes, this will help those students who belong to villages and cannot visit urban ares for studies. II. No, this is not an economically viable proposal, as the number of students who attend such classes cannot contribute to break-even. A. if only argument I is strong. B. if only argument II is strong. C. if either I or II is strong. D. If both I and II are strong. Answer : D. If both I and II are strong. Explanation : ✓ Statement I: The basic purpose of conducting classes is to help students. Hence, statement I is a strong argument as it conveys this idea. ✓ Statement II: If the institute conducts classes with the intention of making profits, then this is a valid point to be considered. Hence, statement II is also strong. 5. Are there any good politicians left in this world? Arguments: I. Yes, so many poor people are sustaining themselves and improving economically. II. No, there is nothing in this world which is completely good or completely bad. A. if only argument I is strong. B. if only argument II is strong. C. if neither I nor II is strong. D. if either I or II is strong. Answer : C. if neither I nor II is strong.

50

Logical Reasoning

Nithra

Explanation : Just because some poor people are improving economically, we cannot conclude that there are good politicians. Any reason could be ascribed to such an improvement. Statement II is very vague and is not giving any reasons for the argument that it proposes. Hence, both the statements are weak arguments. 6. Should there be an upper age limit for a person to assume the post of prime minister? Arguments: I. Yes, a leader must not only be wise and experienced but also be energetic and young enough to understand the trends of the world. II. No, older the leader, better he is, as he can lead more effectively owing to his experience. A. if both I and II are strong. B. if only argument II is strong. C. if either I or II is strong. D. if neither I nor II is strong. Answer : A. if both I and II are strong. Explanation : Statement I says that the prime minister must be "young enough" for some explained reasons. This implies that the prime minister must not be older than a certain age limit. This statement is strong. Statement II: This says that there should not be an upper limit because older persons can perform better because of the experience gained. This is also strong.

51

Logical Reasoning

Nithra

7. Should there be a law to punish parents who get their minor children married? Arguments: I. Yes, a minor girl is physiologically not prepared to conceive a baby. II. No, this has been a custom prevailing since many centuries. A. if only argument I is strong. B. if only argument II is strong. C. if either I or II is strong. D. if neither I nor II is strong. E. If both I and II are strong. Answer : A. if only argument I is strong. Explanation : Statement I explain a valid reason to go against such marriages because it is wrong to encourage anything that jeopardises the health of a person. Hence, (I) is a strong argument. Statement II is not a valid argument because a custom being followed since a long time does not necessarily make it good. Only I is strong. 8. Should the institute conduct classes in remote villages? Arguments: I. Yes, this will help those students who belong to villages and cannot visit urban ares for studies. II. No, this is not an economically viable proposal, as the number of students who attend such classes cannot contribute to break-even. A. if only argument I is strong. B. if only argument II is strong. C. if either I or II is strong. D. if neither I nor II is strong. E. If both I and II are strong.

52

Logical Reasoning

Nithra

Answer : E. If both I and II are strong. Explanation : Statement I: The basic purpose of conducting classes is to help students. Hence, statement I is a strong argument as it conveys this idea. Statement II: If the institute conducts classes with the intention of making profits, then this is a valid point to be considered. Hence, statement II is also strong. 9. Are there any good politicians left in this world? Arguments: I. Yes, so many poor people are sustaining themselves and improving economically. II. No, there is nothing in this world which is completely good or completely bad. A. if only argument I is strong. B. if only argument II is strong. C. if either I or II is strong. D. if neither I nor II is strong. E. if both I and II are strong. Answer : D. if neither I nor II is strong. Explanation : Just because some poor people are improving economically, we cannot conclude that there are good politicians. Any reason could be ascribed to such an improvement. Statement II is very vague and is not giving any reasons for the argument that it proposes. Hence, both the statements are weak arguments.

53

Logical Reasoning

Nithra

10. Are these sanctuaries, which are meant to protect the endangered animals, necessary? Arguments: I. Yes, these are necessary as it is our responsibility to conserve environment and to provide posterity with a better world to live in. II. No, these are a huge burden on our receding economy. A. if only argument I is strong. B. if only argument II is strong. C. if either I or II is strong. D. if neither I nor II is strong. E. if both I and II are strong. Answer : E. if both I and II are strong. Explanation : Sanctuary for endangered animals becomes necessary when it improves and betters the living conditions of the world. But at the same time, if an economy is not able to provide basic necessities to its people, then it cannot think of constructing sanctuaries. Therefore, the decision is based on both the factors viz. requirement and funds supply. Hence, both I and II are strong arguments.

Questions for Practice 1. Should the teachers be stopped from beating the students? Arguments: I. Yes, child psychologies say that beating hinders the learning process in a child. II. No, spare the cane and spoil the child. A. if only argument I is strong. B. if only argument II is strong. C. if either I or II is strong. D. if neither I nor II is strong. E. if both I and II are strong. 54

Logical Reasoning

Nithra

2. Should there be an upper age limit for a person to assume the post of prime minister? Arguments: I. Yes, a leader must not only be wise and experienced but also be energetic and young enough to understand the trends of the world. II. No, older the leader, better he is, as he can lead more effectively owing to his experience. A. if only argument I is strong. B. if only argument II is strong. C. if either I or II is strong. D. if neither I nor II is strong. E. if both I and II are strong. 3. Should Simons, a hardware company, enter the software industry? Arguments: I. Yes, if there is an indigenous software development wing, the Research and Development of hardware would be more effective. II. No, software industry is in recession. A. if only argument I is strong. B. if only argument II is strong. C. if either I or II is strong. D. if neither I nor II is strong. E. if both I and II are strong. 4. It is wise on the part of the government to disinvest from PSUs? Arguments: I. Yes, overheads of PSUs are increasing making the total cost of production higher and higher. II. No, the fundamental purpose of PSUs is to provide employment to the people. A. if only argument I is strong. B. if only argument II is strong. C. if either I or II is strong. 55

Logical Reasoning

Nithra

D. if neither I nor II is strong. E. if both I and II are strong. 5. Should the manufacturing of leather products be prohibited? Arguments: I. Yes, killing animals for any reason other than survivals is making the environment dangerous. II. NO, leather products are the biggest foreign exchange earners. A. if only argument I is strong. B. if only argument II is strong. C. if either I or II is strong. D. if neither I nor II is strong. E. if both I and II are strong.

Input and Output Solved Problems Study the following information carefully and answer the given questions: A word and number arrangement machine when given an input line of words and numbers rearranges them following a particular rule in each step. The following is an illustration of input and rearrangement. Input : 51 pour 32 start now 23 46 house Step I : 23 51 pour 32 start now 46 house Step II : 23 start 51 pour 32 now 46 house Step III : 23 start 32 51 pour now 46 house Step IV : 23 start 32 pour 51 now 46 house Step V : 23 start 32 pour 46 51 now house Step VI : 23 start 32 pour 46 now 51 house and step VI is the last step of the rearrangement As per the rules followed in the above steps, find out in each of the following questions the appropriate steps for the given input. 56

Logical Reasoning

Nithra

1. Step II of an input is: 18 tasks bear cold dish 81 63 31 How many more steps will be required to complete the rearrangement? Answer: Five Explanation: In step I the least number comes to the left most position, pushing the rest of the line rightward. In step II the word that comes last in the alphabetical order shifts to second from left, pushing again the rest of the line rightward. Similarly, in step III the second least number shifts to third from left. In step IV the second from last in the alphabetical order comes to the fourth position. And this goes on alternately till all the numbers are arranged in ascending order and the words in reverse alphabetical order. ✓ ✓ ✓ ✓ ✓ ✓

Step II : 18 task bear cold dish 81 63 31 Step III : 18 task 31 bear cold dish 81 63 Step IV : 18 task 31 dish bear cold 81 63 Step V : 18 task 31 dish 63 bear cold 81 Step VI : 18 task 31 dish 63 cold bear 81 Step VII : 18 task 31 dish 63 cold 81 bear

57

Logical Reasoning

Nithra

2. Input : 72 59 37 go for picnic 24 journey How many steps will it take to complete the rearrangement? Answer: Six Explanation: ✓ ✓ ✓ ✓ ✓ ✓ ✓

Input : 72 59 37 go for picnic 24 journey Step I : 24 72 59 37 go for picnic journey Step II : 24 picnic 72 59 37 go for journey Step III : 24 picnic 37 72 59 go for journey Step IV : 24 picnic 37 journey 72 59 go for Step V : 24 picnic 37 journey 59 72 go for Step VI : 24 picnic 37 journey 59 go 72 for

3. Input : nice flower 24 12 costly height 41 56 Which of the following will be step III? Answer: 12 nice 34 height flower costly 41 56 Explanation: ✓ Step I : 12 nice flower 34 costly height 41 56 ✓ Step II : 12 nice 34 flower costly height 41 56 ✓ Step III : 12 nice 34 height flower costly 41 56

58

Logical Reasoning

Nithra

4. Step II of an input is : 16 victory 19 36 53 store lake town. Which of the following will be step V ? Answer: There will be no such step Explanation: ✓ ✓ ✓ ✓

Step II : 16 victory 19 36 53 store lake town Step III : 16 victory 19 town 36 53 store lake Step IV : 16 victory 19 town 36 store 53 lake Since the line is already arranged, there will be no fifth step.

5. Input: orange 36 59 yellow 41 exam test 12 lemon 85 Which of the following combinations represent the first two and last two elements in the step V of the given input? Answer: 12, lemon and 36, exam Explanation: ✓ Step I: 85 orange 36 59 41 exam test 12 lemon yellow ✓ Step II: test 85 orange 36 41 exam 12 lemon yellow 59 ✓ Step III: 41 test 85 36 exam 12 lemon yellow 59 orange ✓ Step IV: lemon 41 test 85 exam 12 yellow 59 orange 36 ✓ Step V: 12 lemon 41 test 85 yellow 59 orange 36 exam

59

Logical Reasoning

Nithra

6. Input: orange 36 59 yellow 41 exam test 12 lemon 85 Which element comes exactly between 85 and lake in Step III of the given input? Answer: exam 7. Input: orange 36 59 yellow 41 exam test 12 lemon 85 Which of the following element is fourth to the right of the one which is ninth from the right end in step V of the given input? Answer: yellow 8. Input: Ant 43 17 Eagle 24 Forest Gun 25 81 House Which of the following combinations represent the first two and the last two elements in Step IV of the given input? Answer: House Gun and 42 52 9. Input: Ant 43 17 Eagle 24 Forest Gun 25 81 House If in the second step,”17″ interchanges its position with ’25’ and ‘Gun’ also interchanges its position with “Forest” then which element will be the second from the left end? Answer: Forest 10. Input: Ant 43 17 Eagle 24 Forest Gun 25 81 House Which element is third to the left of the one which is eighth from the left in Step III of the given Input? Answer: 17

60

Logical Reasoning

Nithra

Questions for practice 1. Input: orange 36 59 yellow 41 exam test 12 lemon 85 Which of the following combinations represent the first two and last two elements in the step V of the given input? (A) 12, lemon and 36, exam (B) 41, test and orange, 36 (C) lemon, 41 and 59, orange (D) 12, lemon and orange, 36 (E) lemon, 41 and 36, exam 2. Input: table 63 tour 19 typhoon 72 25 to tea 48 tablet 56 Which of the following combinations represent the sixth and eighth element in Step II of the given input from left end? (A) tea and 63 (B) 63 and tablet (C) 48 and table (D) 72 and tablet (E) 72 and tea

61

Logical Reasoning

Nithra

3. Input : Rotator 9 Wow 14 18 Orange 11 Radar 3 Unit 9 Which element is fifth to the right of the one which is eighth from the right in Step II of the given Input? (A) 14 (B) Radar (C) 11 (D) Union (E) Rotator 4. Input: Ant 43 17 Eagle 24 Forest Gun 25 81 House Which of the following combinations represent the first two and the last two elements in Step IV of the given input? (A) House Gun and 34 42 (B) House Gun and 52 71 (C) House Gun and 42 71 (D) House Gun and 34 52 (E) House Gun and 42 52

62

Logical Reasoning

Nithra

5. INPUT: emit random 23 violence 19 speaker 39 expert 33 award 28 Which of the following would be at the ninth position from the left end in step 6? (A) 33 (B) violence (C) expert (D) 39 (E) 28

Letter in word puzzle Solved Problems 1. If the letters in each of the following five groups of letters are first rearranged in the alphabetical order and then the groups of letters so formed are rearranged as in a dictionary, which letter group would have its group of letters in the ''MIDDLE'' among the five letter groups? MEET, DEAF, ROAD, CODE, LACK Answer : ROAD Explanation : Given words are: MEET, DEAF, ROAD, CODE, LACK The letters of each word in alphabetical order are: MEET --- EEMT DEAF --- ADEF ROAD --- ADOR CODE --- CDEO LACK --- ACKL Now, groups of letters so formed, are rearranged as in a dictionary: 63

Logical Reasoning

Nithra

ACKL ----> ADEF ---> ADOR ---> CDEO ---> EEMT Here, the letter group would have its group of letters in the ''MIDDLE'' (i.e., ADOR) is ''ROAD''. 2. If the letters of the word 'PROTECTION' which are at odd numbered position in the English alphabet are picked up and are arranged in alphabetical order from left and if they are now substituted by Z, Y, X and so on, beginning from left which letter will get substituted by X? Answer : I Explanation : Given word: P R O T E C T I O N Position of these letters in the English alphabet: P -- 16, R -- 18, O -- 15, T -- 20, E -- 5, C -- 3, T -- 20, I -- 9, O -- 15, N -- 14 Letters at odd numbered position = O, E, C, I, O Letters at odd numbered position in alphabetical order = C E I O O If they are now substituted by Z, Y, X, W, V, the letter ''I'' will get substituted by X. 3. If each alternate letter in the word 'MENTION' starting with M is changed to the next letter in the English alphabetical series and each of the remaining letters is changed to the previous letter in the English alphabetical series, then how many letters will appear more than once in the new arrangement? Answer : Two Explanation : According to given instruction, new arrangement is: M + 1 position = N E - 1 position = D N + 1 position = O T - 1 position = S I + 1 position = J O - 1 position = N 64

Logical Reasoning

Nithra

N + 1 position = O Thus the new arrangement is ''NDOSJNO''. Here, two letters ''N'' and ''O'' appears twice. 4. If it is possible to make only one meaningful word with the second, fifth, eighth and the eleventh letters (when counted from left hand side) of the word 'DEVELOPMENT', which would be the second letter of the word from the left? If more than one such word can be formed, give X as the answer. If no such word can be formed, give Z as your answer. Answer : E Explanation : Given word: D E V E L O P M E N T Positions : 1 2 3 4 5 6 7 8 9 10 11 2nd, 5th, 8th, 11th letters are : E L M T Only one meaningful word which can be formed with the letters ''ELMT'' is ''MELT'' In the word ''MELT'', second letter from the left = E. 5. How many such pairs of letters are there in the word 'RAZORS', each of which has as many letters between them as in the English alphabet series? Answer : Two Explanation : In the alphabet series, there are 2 letters between 'O' and 'R'. Here also there are 2 letters 'AZ' between them. Again, there is no letter between 'R' and 'S'.

65

Logical Reasoning

Nithra

Questions for practice 1. The positions of first and fifth letters of the word “SEMANTIC” are interchanged. Similarly, the position of second and sixth and so on till the fourth and eight letters are interchanged. In the new arrangement thus formed, how many letters are there in between the letter which is third form the right and third from the left in the English alphabetical order? a) Three

b) Four

c) Two

d) Six

e) Eight

2. In the word 'DOORSTEP' if all the vowels are replaced with the letter immediate next to it in the English alphabet series and all the consonants are replaced with the letter immediate before it in the English alphabet series, then how many vowels are there in the new word so formed? a) None

b) One

c) Two

d) More than two

e)None of these

3. If it is possible to make only one meaningful English word from the second, the fifth, the seventh and the eighth letters of the word BRILLIANT, using each letter only once, second letter of that word is your answer. If more than one such word can be formed, your answer is X. If no such word can be formed, your answer is Y? a) L

b) X

c) R

d) Y

e) A

4. In the word 'EVERYTHING' all consonants are written as their preceding letter and all vowels are written as their succeeding letters. Now all letters are arranged in alphabetical order from left to right and all the repeated letters are eliminated. How many letters are between U and G? a) One

b)Two

c)Four

d)Five

66

e)None

Logical Reasoning

Nithra

5. If it is possible to make a meaningful word with the 2nd, the 6th, the 9th and the 12th letters of the word 'CONTRIBUTION', which of the following will be the last letter of that word? If more than one such word can be formed, give 'M' as the answer and if no such word is there, give 'X' as the answer. a) T

b)O

c)M

d)X

e)N

Mathematical Operations Solved Problems 1. If - means *, * means +, + means / and / means -, then 40 * 12 + 3 - 6 / 60 = ? Answer : 4 Explanation : Using the correct symbols, we have: Given expression: 40 + 12 / 3 * 6 - 60 = 40 + 4 * 6 - 60 = 40 + 24 - 60 = 4. 2. If + means /, * means -, / means * and - means +, than 16 + 8 * 4 / 3 - 4 = ? Answer : -6 Explanation : Using the correct Symbol, we have: Given expression: 16 / 8 - 4 * 3 + 4 = 2 - 4 * 3 + 4 = 2 - 12 + 4 = - 6.

67

Logical Reasoning

Nithra

3. Find out the two signs to be interchanged for making following equation correct: 5 + 3 * 8 - 12 / 4 = 3 Answer : - and / Explanation : we get the equation as 5 + 3 * 8 / 12 - 4 = 3 5 + 3 * 2/3 - 4 = 3 3 = 3, which is true.. 4. Give interchanges : Signs - and * and numbers 3 and 6. 1. 6 - 3 * 2 = 9 2. 3 - 6 * 8 = 10 3. 6 * 3 - 4 = 15 4. 3 * 6 - 4 = 33 Answer : 2. 3 - 6 * 8 = 10 Explanation : On interchanging - and * and 3 and 6 in (2), we get the equation as 6 * 3 - 8 = 10 18 - 8 = 10 10 = 10, which is true.

68

Logical Reasoning

Nithra

5. Select the correct set of symbols which will fit in the given equation? 5 0 3 5 = 20 1. *, *, * 2. -, +, * 3. *, +, * 4. +, -, * Answer : 2. -, +, * Explanation : Clearly 5 - 0 + 3 * 5 = 5 + 15 = 20. 6. If + is * , - is + , * is / and / is - , then answer the following questions based on this information. 9/5+4-32*2=? Answer : 5 Explanation : Using the correct symbols, we have given expression: 9 / 5 + 4 - 3 2 * 2 = 9 - 5 * 4 + 32 / 2 = 9 - 20 + 16 = 25 - 20 = 5.

69

Logical Reasoning

Nithra

7. Given intercharges : Signs - and / and numbers 4 and 8. 1. 6 - 8 / 4 = - 1 2. 8 - 6 / 4 = 1 3. 4 / 8 - 2 = 6 4. 4 - 8 / 6 = 2

Answer : 4 / 8 - 2 = 6 Explanation : On interchanging - and / and 4 and 8 in (3), we get the equation as 8-4/2=6 8-2=6 6 = 6, which is true. 8. If * stands for 'addition', / stands for 'subtraction', + stands for 'multiplication' and-stands for 'division', then 20 * 8 / 8 - 4 + 2 = ? Answer : 24 Explanation : By the Given data , We have the expression: 20 + 8 - 8 / 4 * 2 = 20 + 8 - 2 * 2 = 20 + 8 - 4 = 24.

70

Logical Reasoning

Nithra

9. Find out the two signs to be interchanged for making following equation correct 5 + 3 x 8 - 12 / 4 =3 Answer : - and / Explanation : On interchanging - and /, we get the equation as 5 + 3 x 8 / 12 - 4 = 3 or 5 + 3 x (2/3) - 4 = 3 or 3 = 3, which is true. 10. Given intercharges : Signs + and * and numbers 4 and 5. Answer : 5 * 4 + 20 = 104 Explanation : On interchanging + and x and 4 and 5 in (c), we get the equation as 4 + 5 x 20 = 104 or 104 = 104, which is true.

71

Logical Reasoning

Nithra

Questions for practice 1. If Q means 'add to', J means 'multiply by', T means 'subtract from' and K means 'divide by' then 30 K 2 Q 3 J 6 T 5 = ? A) 18

B) 28

C) 31

D) 103

2. If × means ÷, - means ×, ÷ means + and + means- than (3 - 15 ÷ 19) × 8 + 6 = ? A) 8

B) 4

C) 2

D) -1

3. In a certain code language, '÷' represents '+', '-' represents 'x', '+' represents '÷' and 'x' represents '-'. Find out the answer to the following question. 8 ÷ 14 - 20 + 5 x 2=? A) 46

B) 62

C) 32

D) 14

4. The following equation is incorrect. Which two signs should be interchanged to correct the equation? 1 ÷ 14 x 30 + 20 - 10 = 12 A) - and +

B) ÷ and x

C) x and -

D) + and ÷

5. In a certain code language, '-' represents 'x', '÷' represents '+', '+' represents '÷' and 'x' represents '-'. Find out the answer to the following question.19 ÷ 2 - 35 + 10 x6=? A) 20

B) 14

C) 32

D) 16

72

Logical Reasoning

Nithra

Sentence Completion Solved Problems 1. The goal of any company is to have its product name become ______constantly at the forefront of the consumer's mind. A. garrulous B. unctuous C. tremulous D. ubiquitous Answer : D. ubiquitous Explanation : Ubiquitous (adj.) means being present everywhere. 2. Neem has ______ qualities and in many clinical trials, doctor have saved countless lives by using raw Neem leaves on serious wounds. A. remedial B. inferior C. doubtful D. notorious Answer : A. remedial Explanation : We need a positive word with a 'life' saving-like meaning. Choices B, C, and D are negative and are ruled out. You may be unaware of remedial. Therefore, the answer is remedial.

73

Logical Reasoning

Nithra

3. During the holiday season, the ______ theme is Peace on Earth, Good will Toward All. A. mitigated B. arrogant C. controversial D. prevalent Answer : D. prevalent Explanation : Prevalent (adj.) means widespread or widely accepted; predominant or extensive. 4. Lynette had to learn the ______ of the insurance profession before she felt comfortable describing products to her clients. A. lexicon B. classicism C. juncture D. cessation Answer : A. lexicon Explanation : Lexicon is the vocabulary used in a language, profession, class, or subject.

74

Logical Reasoning

Nithra

5. Philosophical problems arise when people and questions that, though very_______, have certain characteristics in common. A. relevant B. elementary C. abstract D. diverse Answer : D. diverse Explanation : The key to the correct choice is in the word though, which indicates that the missing word must convey a sense which is the opposite of the phrase 'having certain characteristics in common', Among the choices, diverse (meaning various of differing in qualities), is the most suitable answer. The sentence now means that though people ask diverse questions, there are some common characteristics among them. 6. Far from being mere replicas of seventeenth-century African culture, Maroon societies have continually developed as their members have________________the artistic heritage bequeathed by their ancestors, adapting it creatively to their changing lives. A. confused B. repressed C. denied D. modified Answer : D. modified Explanation : We can surmise from the sentence that Maroon societies are African groups. The sentence says that their culture has not remained static at the seventeenthcentury level, but has been adapted creatively to their changing lives. The missing 75

Logical Reasoning

Nithra

verb must therefore be a synonym of this italicized phrase. Among the choices, it is modified which has a similar meaning and is the answer. 7. For silent movies to succeed, the quality of acting has to be___ as there are no ______ dialogues or music for support. A. Mediocre...engaging B. Sublime...intimate C. Realistic...melodramatic D. Substantial...vehement E. Extraordinary...mesmerizing Answer : E. Extraordinary...mesmerizing Explanation : Key words: for support. Both the blanks must take positive and strong words. 8. Every minister must be made_______ to the public for his/her acts of omission and commission. A. menacing B. acceptable C. approachabler D. accountable E. dispensable Answer : D. Accountable Explanation : Every minister must be made¬ accountable to the public for his/her acts of omission and commission. Accountable means to be responsible for one’s decisions or actions.

76

Logical Reasoning

Nithra

Menacing means threatening. Acceptable and approachable do not fit the context. 9. Suresh is a _________ who can dupe even the not so gullible. A. smooth operator B. perfect gentleman C. knowledgeable person D. jack of all trades E. blabbermouth Answer : A. smooth operator Explanation : Suresh is a smooth operator who can dupe even the not so gullible. The key part of the sentence is the expression ‘even the not so gullible’, which indicates that a negative word or expression will fit the blank. 10. At a time when most charities seem to think of _____ increasing their capital, Warren Buffet’s stipulation that what he gifts must be spent within ten years comes as a _______ announcement. A. perennially ..... contradictory B. merely.....surprise C. eternally ..... Involuntary D. sporadically.....refreshing E. constantly ..... landmark Answer : E. constantly ..... landmark

77

Logical Reasoning

Nithra

Explanation : Though constantly, perennially and eternally are synonyms yet constantly fits the blank best as it simply means continuously, whereas perennially and eternally mean everlasting and are too strong for the given context. A landmark announcement is an important announcement.

Questions for practice 1. Challenges must be __________ to realize the _________ of a greater regional economic integration. a) Overcome....Potential b) Suppressed....power c) Ignored....benefits d) Sustained....Advantages e) Attempted....battles 2. He’s got a sharp ________. He might just get into trouble, if he isn’t careful. a) Tongue b) Mouth c) Intellect d) Vision e) Brain 78

Logical Reasoning

Nithra

3. We had lunch _____ a Chinese restaurant yesterday a) In b) On c) At d) Inside e) Around 4. Vinod is clearly _________ when it comes to speaking English. a) To a disadvantage b)At a disadvantage c)From a disadvantage d)In a disadvantage e)On a disadvantage 5. It was clear that there was no rationale behind his act; he went solely by his _____ and somehow succeeded. a) Logic b)Interpretation c) Intuition d)Analysis

79

Logical Reasoning

Nithra

e)Compass

Sequence of words Solved Problems 1. Arrange the words given below in a meaningful sequence 1. Gold 2. Iron 3. Sand 4. Platinum 5. Diamond A. 2, 4, 3, 6, 1 B. 3, 2, 1, 5, 4 C. 4, 5, 1, 3, 2 D. 5, 4, 3, 2, 1 Answer: B. 3, 2, 1, 5, 4 Explanation: The correct order is : Sand, Iron, Gold, Diamond, Platinum. 2. Arrange the words given below in a meaningful sequence. 1. Cut 2. Put on 3. Mark 4. Measure 5. Tailor A. 4, 3, 1, 5, 2 B. 3, 1, 5, 4, 2 C. 2, 4, 3, 1, 5 80

Logical Reasoning

Nithra

D. 1, 3, 2, 4, 5 Answer: A. 4, 3, 1, 5, 2 Explanation: The correct order is : Measure, Mark, Cut, Tailor, Put on. 3. Arrange the given words in a meaningful sequence. 1. Nation 2. Village 3. City 4. District 5. State A. 2, 3, 4, 5, 1. B. 2, 3, 4, 1, 5. C. 1, 3, 5, 4, 2. D. 1, 2, 3, 4, 5. Answer: A. 2, 3, 4, 5, 1. Explanation: The correct order is : Village, City, District, State, Nation. 4. Arrange the words given below in a meaningful sequence. 1. Index 2. Contents 3. Title 4. Chapters 5. Introduction

81

Logical Reasoning

Nithra

A. 2, 3, 4, 5, 1 B. 3, 2, 5, 1, 4 C. 3, 2, 5, 4, 1 D. 5, 1, 4, 2, 3 Answer: C. 3, 2, 5, 4, 1 Explanation: The correct order is : Title, Contents, Introduction, Chapters, Index. 5. Arrange the words given below in a meaningful sequence. 1. Puberty 2. Adulthood 3. Childhood 4. Infancy 5. Senescence 6. Adolescence A. 2, 4, 6, 3, 1, 5 B. 4, 3, 1, 6, 2, 5 C. 4, 3, 6, 2, 1, 5 D. 5, 6, 2, 3, 4, 1 Answer: 4, 3, 1, 6, 2, 5 Explanation: The correct order is : Infancy, Childhood, Puberty, Adolescence, Adulthood, Senescence.

82

Logical Reasoning

Nithra

6. Arrange the words given below in a meaningful sequence. 1. Windows 2. Walls 3. Floor 4. Foundation 5. Roof 6. Room A. 4, 1, 5, 6, 2, 3 B. 4, 2, 1, 5, 3, 6 C. 4, 3, 5, 6, 2, 1 D. 4, 5, 3, 2, 1, 6 Answer: B. 4, 2, 1, 5, 3, 6 Explanation: The correct order is : Foundation, Walls, Windows, Roof, Floor, Room. 7. Arrange the words given below in a meaningful sequence. 1. Probation 2. Interview 3. Selection 4. Appointment 5. Advertisement 6. Application A. 5, 6, 2, 3, 4, 1 B. 5, 6, 3, 2, 4, 1 C. 5, 6, 4, 2, 3, 1 D. 6, 5, 4, 2, 3, 1 Answer: A. 5, 6, 2, 3, 4, 1

83

Logical Reasoning

Nithra

Explanation: The correct order is : Advertisement, Application, Interview, Selection, Appointment, Probation. 8. Arrange the words given below in a meaningful sequence. 1. Word 2. Paragraph 3. Sentence 4. Letters 5. Phrase A. 4, 1, 5, 2, 3 B. 4, 1, 5, 3, 2 C. 4, 2, 5, 1, 3 D. 4, 1, 3, 5, 2 Answer : B. 4, 1, 5, 3, 2 Explanation : The correct order is :Letters - 4, Word - 1, Phrase - 5, Sentence - 3, Paragraph – 2 9. Arrange the following words in a logical sequence. 1. Grass 2. Curd 3. Milk 4. Cow 5. Butter A. 4, 1, 3, 2, 5 B. 2, 3, 4, 5, 1 C. 1, 2, 3, 4, 5 D. 5, 4, 3, 2, 1 Answer : A. 4, 1, 3, 2, 5 Explanation : We know that cow eats grass and then gives milk. With the milk, curd is made and then from curd, butter is made.

84

Logical Reasoning

Nithra

Hence logical sequence is Cow, Grass, Milk, Curd, Butter. 10.

Arrange

the

1.Protect

following

2.Pressure

words

in

an

3.Relief

appropriate 4.Rain

order 5.Flood

A).2,4,5,1,3 B).2,4,3,1,5 C).2,5,4,1,3 D).3,2,4,5,1 Answer:A).2,4,5,1,3 Explanation

:

The Correct Sequence is: Pressure - 2, Rain - 4, Flood - 5, Protect - 1, Relief - 3.

Questions for Practice 1. Arrange the words given below in a meaningful sequence. 1. Key 2. Door 3. Lock 4. Room 5. Switch on A. B. C. D.

5, 1, 2, 4, 3 4, 2, 1, 5, 3 1, 3, 2, 4, 5 1, 2, 3, 5, 4

2. Arrange the words given below in a meaningful sequence. 1. Word 2. Paragraph 3. Sentence 4. Letters 5. Phrase A. B. C. D.

4, 1, 5, 2, 3 4, 1, 3, 5, 2 4, 2, 5, 1, 3 4, 1, 5, 3, 2 85

Logical Reasoning

Nithra

3. Arrange the words given below in a meaningful sequence. 1. Police 2. Punishment 3. Crime 4. Judge A. B. C. D.

5. Judgement

3, 1, 2, 4, 5 1, 2, 4, 3, 5 5, 4, 3, 2, 1 3, 1, 4, 5, 2

4. Arrange the words given below in a meaningful sequence. 1. Family 2. Community 3. Member 4. Locality 5. Country A. B. C. D.

3, 1, 2, 4, 5 3, 1, 2, 5, 4 3, 1, 4, 2, 5 3, 1, 4, 5, 2

5. Arrange the words given below in a meaningful sequence. 1. Poverty 2. Population 3. Death 4. Unemployment A. B. C. D.

5. Disease

2, 3, 4, 5, 1 3, 4, 2, 5, 1 2, 4, 1, 5, 3 1, 2, 3, 4, 5

Situation Reaction Test Solved Problems Direction: To each of the following question four probable answers have been given. select the most appropriate alternative as the answer.

86

Logical Reasoning

Nithra

1. You are walking down the street and suddenly you see two hundred rupee notes on the pavement. What action will you take? A. Deposite it in the nearest police station. B. Leave it where it is. C. Give the money to a beggar. D. Pocket it yourself. Answer : A. Deposite it in the nearest police station. Explanation : As the money lying on the street is not yours, so keeping it with yourself of giving it to a beggar is not ethical. The correct action in this situation is to deposit that money in the nearest police station so that the genuine owner of the money will receive that money. 2. If in the examination hall, you find that the question paper is too tough to be answered satisfactorily by you, the best thing to do for you is to : A. Tell the examiner that the questions are out of course. B. Try to solve the questions as much as you know with a cool head. C. Try to know something from your neighbour. D. Provoke the candidates to walk out of the examination hall. Answer : B. Try to solve the questions as much as you know with a cool head. Explanation : The most appropriate reaction in this situation would be try to solve the questions as much as you know with a cool head.

87

Logical Reasoning

Nithra

3. While firing crackers, a child gets severe burns on the hand. What would you do ? A. Send someone to call the doctor. B. Wash the hands with Dettol. C. Dip the child's hands in cold water till there is no more burning sensation. D. Apply some ointment on the affected area. Answer : C. Dip the child's hands in cold water till there is no more burning sensation. Explanation : The most appropriate reaction in this situation would be Dip the child's hands in cold water till there is no more burning sensation. 4. While travelling in your car, certain persons stop you on the way asking you to take an injured child to the hospital. You would --A. Ask them to leave your way and then drive away B. Immediately take the child to hospital C. Ask them to first call the police D. Get out of the car and some other person to help the child Answer : B. Immediately take the child to hospital Explanation : The most appropriate reaction in this situation would be Immediately take the child to hospital 5. While traveling in a train, you found that some college students pulling the alarm chain simply to get down at their desired point, you would A. Let them pull the chain but chek them from detrain B. with the help of our passengers check them from do C. Inform the 88

Logical Reasoning

Nithra

D. Keep quite and do nothing. Answer : B. with the help of our passengers check them from do Explanation : The most appropriate reaction in this situation would be with the help of our passengers check them from doing so. 6. On reaching the railway station, you find that the train you wanted to catch is just to start and there is hardly any time for purchasing the ticket. The best thing for you is to : A. first purchase the ticket and then catch the train if it is there. B. rush to the train and perform your journey quietly. C. rush to the train rather than miss it and inform the T.T.I. at the next stoppage about your inability to purchase the ticket. D. miss the train rather than take the risk of boarding the moving train. Answer : C. rush to the train rather than miss it and inform the T.T.I. at the next stoppage about your inability to purchase the ticket. Explanation : The most appropriate reaction in this situation would be ush to the train rather than miss it and inform the T.T.I. at the next stoppage about your inability to purchase the ticket. 7. You are in a bus. The bus reaches your stop but still you have not purchased the ticket because of heavy rush. What will you do? A. Call the conductor, give him the money and get the ticket. B. Jump out quickly to avoid embarrassment. C. Hand the money to someone sitting nearby to give it to the conductor. D. Give the money to the driver. 89

Logical Reasoning

Nithra

Answer : A. Call the conductor, give him the money and get the ticket. Explanation : The most appropriate reaction in this situation would be Call the conductor, give him the money and get the ticket. 8. Your standing at the door of the train that has just left platform. Suddenly you see a man from your behind coach falling off the train. What will you do ? A. Wait till the train stops at the next station and B. Shout at the falling man asking him to get up quic C. Jump off the train to assist the falling man. D. Pull the alarm chain so that the train may stop an Answer : D. Pull the alarm chain so that the train may stop an Explanation : The most appropriate reaction in this situation would be Pull the alarm chain so that the train may stop and the man may be helped. 9. Atmosphere always has A. Oxygen B. Air C. Dust D. Moisture Answer : B. Air Explanation : Clearly, though all the alternatives may form a part of the atmosphere, the air is the most vital part, without which there can be no atmosphere. 90

Logical Reasoning

Nithra

10. A factory always has A. Electricity B. Chimney C. Workers D. Files Answer : C. Workers Explanation : Clearly, without workers there can be no factory.

Questions for practice 1. You have made some silly mistakes which have been pointed out to you. You will: A. laugh it away. B. get angry. C. feel miserable. D. feel thankful. 2. You are moving across the road on a scooter when you observe that two boys on a bike snatch a lady's gold chain and ride away. You would : A. Console the woman. B. chase the boys to catch hold of them. C. inform the police about the matter. D. stand and see what happens next.

91

Logical Reasoning

Nithra

3. You are walking down the street and suddenly you see two hundred-rupee notes on the pavement. What action will you take ? A. Pocket it yourself. B. Leave it where it is. C. Give the money to a beggar. D. Deposit it in the nearest police station. 4. Your bathroom tap id leaking and is a constant source of irritating noise. You would : A. sleep with pillows upon your ears. B. put a bucket underneath. C. try to put up a cork upon the mouth of the tap. D. call a plumber to repair the tap. 5. You find a lady's purse dropped on the road and on picking it up, find a thousand rupees inside. You would : A. take the purse away. B. take out the money and leave the purse there. C. deposit it at the nearest police station. D. stand there and wait for the owner.

Statement and Assumption Solved Problems In each question below is given a statement followed by three assumptions numbered I, II and III. You have to consider the statement and the following assumptions, decide which of the assumptions is implicit in the statement and choose your answer accordingly.

92

Logical Reasoning

Nithra

1. Statement: 'To make the company commercially viable, there is an urgent need to prune the staff strength and borrow money from the financial institutions.' Opinion of a consultant. Assumptions: I. The financial institutions lend money for such proposals. II. The product of the company has a potential market. III. The employees of the company are inefficient. A. None is implicit B. Only I and II are implicit C. All are implicit D. Only II and III are implicit E. Only I and III are implicit Answer: B. Only I and II are implicit Explanation: Since the consultant talks of borrowing money from financial institutions, so I is implicit. That the owners wish to make the company 'commercially viable' makes II implicit. Also, it is mentioned that staff strength is to be reduced to make the company 'commercially viable' So, III is not implicit.

93

Logical Reasoning

Nithra

2. Statement: State Council For Teacher Education (SCTE) has laid down guidelines in respect of minimum qualifications for a person to be employed as a teacher in universities or in recognised institutions. Assumptions: I. The authorities will now appoint only qualified teachers. II. Only qualified people will apply for the teaching post. III. SCTE decides all the norms of educational qualifications for teaching faculty. A. None is implicit B. Only I is implicit C. All are implicit D. Only I and III are implicit E. Only I and II are implicit Answer: C. All are implicit Explanation: Since the SCTE has laid down the necessary qualifications for a person to be employed as a teacher in all universities and institutions. So all are implicit. 3. Statement: A State Government suspended two additional district judges. Assumptions: I. They were negligent in discharging duties. II. There was a charge of misconduct against them. III. The government officials were biased against them. A. None is implicit B. Any one of the three is implicit C. Either I or II is implicit D. Only I and III are implicit E. Either I or III is implicit

94

Logical Reasoning

Nithra

Answer: C. Either I or II is implicit Explanation: A person holding an office is generally suspended on charges of misconduct or negligence of duty. So, either I or II is implicit. III seems to be vague and so it is not implicit. 4. Statement: "As there is a great demand, every person seeking tickets of the programme will be given only five tickets." Assumption: I. The organizers are not keen on selling the tickets. II. No one is interested in getting more than five tickets. A. Only assumption I is implicit B. Only assumption II is implicit C. Neither I nor II is implicit D. Either I or II is implicit Answer : C. Neither I nor II is implicit Explanation : Clearly, the organisers are adopting this policy not to reduce the sale but to cope up with great demand so that everyone can get the ticket. So, I is not implicit. Also, due to great demand, the maximum number of tickets one person can get has been reduced to five. So, II is also not implicit.

95

Logical Reasoning

Nithra

5. Statement : People of all ages have found us good people to grow with. Assumption: I. People want to grow. II. We are good people. A. if only assumption II is implicit. B. if only assumption I is implicit. C. if either I or II is implicit. D. if neither I nor II is implicit. Answer : B. if only assumption I is implicit. Explanation : As in the statement the author says that there are people who are looking for good people to grow with, that means people want to grow. Hence, I is implicit. According to the statement, people found these people to be good, but not the author by himself. Hence, II is not implicit. 6. Statement: Maruti Suzuki is preparing to roll out its compact car, A-star, in mid November, this year in India. A-star will be the eighth model to be launched by Suzuki in less than 40 months. Assumption: I. There is huge demand for new models of the cars in India. II. Suzuki had launched its first car three years ago. A. if only conclusion II follows. B. if only conclusion I follows. C. if either I or II follows. D. if neither I nor II follows. Answer : A. if only conclusion II follows.

96

Logical Reasoning

Nithra

Explanation : There is no clear information given regarding the demand in India. I is not a conclusion. From the statement, A-star will be in less than 40 months it can be concluded that 40 months ago its first car launched. 40 months means 3 years and 4 months. Conclusion II follows. Only II follows. 7. Statement: "You are hereby appointed as a programmer with a probation period of one year and your performance will be reviewed at the end of the period for confirmation." - A line in an appointment letter. Assumption: I. The performance of an individual generally is not known at the time of appointment offer. II. Generally an individual tries to prove his worth in the probation period. A. B. C. D. E.

Only assumption I is implicit Only assumption II is implicit Either I or II is implicit Neither I nor II is implicit Both I and II are implicit

Answer: Option E Explanation: The performance of the individual has to be tested over a span of time as the statement mentions. So, I is implicit. The statement mentions that the individual's worth shall be reviewed (during probation period) before confirmation. So, II is also implicit.

97

Logical Reasoning

Nithra

8. Statement: It is desirable to put the child in school at the age of 5 or so. Assumptions: I. At that age the child reaches appropriate level of development and is ready to learn. II. The schools do not admit children after six years of age. A. B. C. D. E.

Only assumption I is implicit Only assumption II is implicit Either I or II is implicit Neither I nor II is implicit Both I and II are implicit

Answer: Option A Explanation: Since the statement talks of putting the child in school at the age of 5, it means that the child is mentally prepared for the same at this age. So, I is implicit. But nothing about admission after 6 years of age is mentioned in the statement. So, II is not implicit. 9. Statement: "In order to bring punctuality in our office, we must provide conveyance allowance to our employees." - In charge of a company tells Personnel Manager. Assumptions: I. Conveyance allowance will not help in bringing punctuality. II. Discipline and reward should always go hand in hand. A. B. C. D. E.

Only assumption I is implicit Only assumption II is implicit Either I or II is implicit Neither I nor II is implicit Both I and II are implicit

Answer: Option B 98

Logical Reasoning

Nithra

Explanation: Assumption I go against the statement. So, it is not implicit. The allowance will serve as a reward to the employees and shall provoke them to come on time. So, II is implicit. 10. Statement: Unemployment allowance should be given to all unemployed Indian youth above 18 years of age. Assumptions: I. There are unemployed youth in India who needs monetary support. II. The government has sufficient funds to provide allowance to all unemployed youth. A. B. C. D. E.

Only assumption I is implicit Only assumption II is implicit Either I or II is implicit Neither I nor II is implicit Both I and II are implicit

Answer: Option A Explanation: I directly follows from the statement and so is implicit. Also, the statement is a suggestion and does not tell about a government policy or its position of funds. So, II is not implicit.

99

Logical Reasoning

Nithra

Questions for practice 1. Statement: "If you trouble me, I will slap you." - A mother warns her child. Assumptions: I. With the warning, the child may stop troubling her. II. All children are basically naughty. A. B. C. D. E.

Only assumption I is implicit Only assumption II is implicit Either I or II is implicit Neither I nor II is implicit Both I and II are implicit

2. Statement: The State government has decided to appoint four thousand primary school teachers during the next financial year. Assumptions: I. There are enough schools in the state to accommodate four thousand additional primary school teachers. II. The eligible candidates may not be interested to apply as the government may not finally appoint such a large number of primary school teachers. A. B. C. D. E.

Only assumption I is implicit Only assumption II is implicit Either I or II is implicit Neither I nor II is implicit Both I and II are implicit

100

Logical Reasoning

Nithra

3. Statement: A warning in a train compartment - "To stop train, pull chain. Penalty for improper use Rs. 500." Assumptions: I. Some people misuse the alarm chain. II. On certain occasions, people may want to stop a running train. A. B. C. D. E.

Only assumption I is implicit Only assumption II is implicit Either I or II is implicit Neither I nor II is implicit Both I and II are implicit

4. Statement: If it is easy to become an engineer, I don't want to be an engineer. Assumptions: I. An individual aspires to be professional. II. One desires to achieve a thing which is hard earned. A. B. C. D. E.

Only assumption I is implicit Only assumption II is implicit Either I or II is implicit Neither I nor II is implicit Both I and II are implicit

5. Statement: The concession in rail fares for the journey to hill stations has been cancelled because it is not needed for people who can spend their holidays there. Assumptions: I. Railways should give concession only to needy persons. II. Railways should not encourage people to spend their holidays at hill stations. A. B. C. D. E.

Only assumption I is implicit Only assumption II is implicit Either I or II is implicit Neither I nor II is implicit Both I and II are implicit

101

Logical Reasoning

Nithra

Syllogism Solved Problems In each of the questions below three statements are given followed by two conclusions numbered I and II. You have to take the given statements to be true even if they seem to be at variance with commonly known facts. Read all the conclusions and then decide which of the given conclusions logically follows from the given statements, disregarding commonly known facts. 1. Statements: Some buildings are white. All whites are oranges. Some oranges are boxes. Conclusions: I. Some boxes are buildings. II. Some oranges are buildings. Answer: If only conclusion II follows. Explanation: Some buildings are white + All whites are oranges = I + A = I Some buildings are oranges - conversion - some oranges are buildings (I). Hence II follows. Some buildings are oranges + Some oranges are boxes = I + I = No Conclusions. Hence I does not follow.. 2. Statements: All desks are mirrors. All mirrors are chairs. Some chairs are pens. Conclusions: I. Some pens are desks. II. Some chairs are desks. Answer: If only conclusion II follows.

102

Logical Reasoning

Nithra

Explanation: All desks are mirrors + All mirrors are chairs = A + A = A = All desks are chairs - conversion - some chairs are desks (I). Hence II follows. All desks are chairs + some chairs are pens = A + I = No conclusions. Hence I does not follows. 3. Statements: All dollars are rupees. No rupee is yen. No rupee is pound. Conclusions: I. Some pounds are not dollars II. Some dollars are not yens Answer: Both I and II follow Explanation: I. All dollars are rupees + No rupees is pound = All + No = No = No dollars is pound => Some pounds are not dollar. Hence I is true II. All dollars are rupees. No rupee is yen= All+ No = No = No dollars is yen => Some dollars are not yen is true.

103

Logical Reasoning

Nithra

4. Statements: Some glasses are rivers. Some rivers are bottles. Some bottles are plates. Conclusions: I. Some plates are rivers. II. Some bottles are glasses. Answer: If neither conclusion I nor conclusion II follows. Explanation: I + I = No conclusions. 5. Statements: No books is a pen. All pens are pencils. All pencils are erasers. Conclusions: I. All pens are erasers. II. At least some erasers are pencils. Answer: If both conclusions I and II follows. Explanation: All pens are pencils All pencils are erasers. A + A = A - type of Conclusion ''All pens are erasers.'' This is conclusion I. Conclusion II is Converse of the second Premise.

104

Logical Reasoning

Nithra

6. Statements: All the actors are girls. All the girls are beautiful. Conclusions: All the actors are beautiful. Some girls are actors. Answer: Both (1) and (2) follow 7. Statements: All the windows are doors. No door is a wall. Conclusions: Some windows are walls. No wall is a door. Answer: Only (2) conclusion follows 8. Statements: All cups are books. All books are shirts. Conclusions: Some cups are not shirts. Some shirts are cups. Answer: Only (2) conclusion follows 9. Statements: Some cows are crows. Some crows are elephants. Conclusions: Some cows are elephants. All crows are elephants. Answer: Neither (1) nor (2) follows 10. Statements: All the pencils are pens. All the pens are inks. Conclusions: All the pencils are inks. Some inks are pencils. Answer: Both (1) and (2) follow

105

Logical Reasoning

Nithra

Questions for practice In each of the following questions two statements are given and these statements are followed by two conclusions numbered (1) and (2). You have to take the given two statements to be true even if they seem to be at variance from commonly known facts. Read the conclusions and then decide which of the given conclusions logically follows from the two given statements, disregarding commonly known facts. Give answer: (A) If only (1) conclusion follows (B) If only (2) conclusion follows (C) If either (1) or (2) follows (D) If neither (1) nor (2) follows and (E) If both (1) and (2) follow. 1. Statements: Some actors are singers. All the singers are dancers. Conclusions: Some actors are dancers. No singer is actor. A. Only (1) conclusion follows B. Only (2) conclusion follows C. Either (1) or (2) follows D. Neither (1) nor (2) follows E. Both (1) and (2) follow

106

Logical Reasoning

Nithra

2. Statements: All the harmoniums are instruments. All the instruments are flutes. Conclusions: All the flutes are instruments. All the harmoniums are flutes. A. Only (1) conclusion follows B. Only (2) conclusion follows C. Either (1) or (2) follows D. Neither (1) nor (2) follows E. Both (1) and (2) follow 3. Statements: Some mangoes are yellow. Some tixo are mangoes. Conclusions: Some mangoes are green. Tixo is a yellow. A. Only (1) conclusion follows B. Only (2) conclusion follows C. Either (1) or (2) follows D. Neither (1) nor (2) follows E. Both (1) and (2) follow 4. Statements: Some ants are parrots. All the parrots are apples. Conclusions: All the apples are parrots. Some ants are apples. A. Only (1) conclusion follows B. Only (2) conclusion follows C. Either (1) or (2) follows D. Neither (1) nor (2) follows E. Both (1) and (2) follow

107

Logical Reasoning

Nithra

5. Statements: Some papers are pens. All the pencils are pens. Conclusions: Some pens are pencils. Some pens are papers. A. Only (1) conclusion follows B. Only (2) conclusion follows C. Either (1) or (2) follows D. Neither (1) nor (2) follows E. Both (1) and (2) follow

Alphabet test Definitions Alphabet Test is to find the place of an alphabet or a word based on the different types of arrangement.

Solved Problems 1. How many pairs of letters are there in the word " CASTRAPHONE" which have as many letters between them in the word as in the alphabet? Answer: 6 Explanation: Looking into the alphabets there are six such pairs namely ON, HONE, ST, TRAPHO, TRAPHON, RAP. 1. ON - NO 2. HONE - EFGH

108

Logical Reasoning

Nithra

3. ST - ST 4. TRAPHO - OPQRST 5. TRAPHON - NOPQRST 6. RAP – PQR

2. A B C D E F G H I J K L M N O P Q R S T U V W X Y Z . Which letter in this alphabet is the eighth letter to the right of the letter and which is tenth letter to the left of the last but one letter of the alphabet? Answer: W Explanation: In the given alphabet, last but one letter of alphabet is Y. 10th letter to the left of Y is O 8th letter to the right of O is W 3. How many pairs of letters in the word 'CHAIRS' have as many letters between them in the word as in the alphabet? Answer: 2 Explanation: Letters in the word

Letters in the alphabet

CHA

CBA

RS

RS 109

Logical Reasoning

Nithra

4. Arrange these words in alphabetical order and tick the one that comes last 1. Abandon 2. Actuate 3. Accumulate 4. Acquit 5. Achieve Explanation: First letters are common. Second letters are: b, c, c, c, c. One of the four words having c is the last word. Let us see the third letters now, there are: t, c, q, h. Clearly t is the last. Hence Actuate is the last word. 5. What should come next in the following letter sequence? AABABCABCDABCDEABCD Answer: E 6. How many meaningful English words can be formed with the letters ESRO using each letter only once in each word? Answer: Three Explanation: Meaningful words are ROSE, SORE and EROS. Note the meaning of the words SORE and EROS with the help of dictionary. 7. S L U A Y J V E I O N Q G Z B D R H What will come in place of question (?) mark in the following series : LA UJ YI EG ? Answer: QH Explanation: The first letter follows +1, +2, +3, +4, .... The second letter follows +2, +3, +4, +5,....

110

Logical Reasoning

Nithra

8. LAP BUT CAR SON HID If the positions of the first and the third alphabets of each of the words are interchanged, which of the following would form a meaningful word in the new arrangement? Answer: Both LAP and BUT Explanation: When positions of first and the third alphabets of each of the words are interchanged, we have PAL TUB RAC NOS DIH Clearly, PAL and TUB are the only meaningful words. These words are obtained from LAP and BUT respectively. 9. A B C D E F G H I J K L M N O P Q R S T U V W X Y Z. Which letter is in the middle of 13th letter from the left and 4th letter from the right? Answer: R Explanation: Find out the positions of both the letters from the left and divide the sum of the positions of both the letters from the left by 2. Here 13+23=36. Now divide 36 by 2 , i.e 18. Hence R is the required letter.

111

Logical Reasoning

Nithra

10. Q 2 3 B 9 V 5 L S R F P If one is subtracted from each of the numbers, which of the following will be the fourth to the right of the tenth from the right ? Answer: 4 Explanation: Fourth to the right of the tenth from the right means (10 - 4 = )6th from the right, ie 5. But according to the question, one is subtracted from each of the numbers, ie 4.

Questions for Practice 1. C U B A E D E D A B E B A U C D B C A D B D U B C A C B E D A If all the A's are dropped from the above arrangement, which of the following will be eleventh from the left end of the above arrangement? A) E C) D

B) C D) U

2. If the first and second letters in the word DEPRESSION' were interchanged, also the third and the fourth letters, the fifth and the sixth letters and so on, which of the following would be the seventh letter from the right ? A) R C) S

B) O D) P

3. B M N G O P C Q R H S T F L U V W X Y A K Z D I E J Counting from left if it is possible to make a meaningful word from the third and fifth letters from left, using each at least twice and third letter from right in sequence, only once , write the first letter of the word as your answer. If more than 112

Logical Reasoning

Nithra

one such word can be formed write "M" as your answer, and if no word can be formed write 'X' as your answer. A) M B) X C) O D) I 4. A B C D E F G H I J K L M N O P Q R S T U V W X Y Z If the first half of English alphabet is reversed and the second half of the English alphabet is left undisturbed then which letter will be 10th to the left of 16th letter from the left? A) H C) Z

B) F D) U

5. A B C D E F G H I J K L M N O P Q R S T U V W X Y Z If the second half of the English alphabet is reversed then which letter will be 4th to the right of 20th letter from the right? A) U C) E

B) K D) F

Arithmetic Signs Solved Problems 1. Find out the two signs to be interchanged for making following equation correct: 5 + 3 * 8 - 12 / 4 =3 A. + and B. - and / C. + and * D. + and / 113

Logical Reasoning

Nithra

Answer: B. - and / Explanation: On interchanging - and /, we get the equation as 5 + 3 * 8 / 12 - 4 = 3 5 + 3 * (2/3) - 4 = 3 3 = 3, which is true. 2. If 3, 2 = 13; 4, 8 = 80; then 1, 5 =? A. 14 B. 26 C. 25 D. 50 Answer: B. 26 Explanation: The sum of the square of the number 32 + 22 = 9 + 4 = 13 42 + 82 = 16 + 64 = 80 12 + 52 = 1 + 25 = 26.

114

Logical Reasoning

Nithra

3. If 1 = 17 2 = 33 3 = 49 4 = 65 then 5 = ? A. 80 B. 81 C. 89 D. 84 Answer: B. 81 Explanation: Multiply by 16 and add 1 (1 * 16) + 1 = 17 (2 * 16) + 1 = 33 (3 * 16) + 1 = 49 (4 * 16) + 1 = 65 (5 * 16) + 1 = 81 4. If 2 + 3 = 13 3 + 4 = 25 4 + 5 = 41 then 5+6=? A. 60 B. 61 C. 51 D. 63 Answer: B. 61

115

Logical Reasoning

Nithra

Explanation: Step 1 : First multiply to the given number Step 2 : Multiply by 2 Step 3 : add 1 (2 × 3) × 2 + 1 = 13 (3 × 4) × 2 + 1 = 25 (4 × 5) × 2 + 1 = 41 (5 × 6) × 2 + 1 = 61. 5. Select the correct set of symbols which will fit in the given equation? 5 0 3 5 = 20 A. × , ×, × B. -, +, × C. ×, + , × D. +, -, × Answer: B. -, +, × Explanation: Clearly 5 - 0 + 3 × 5 = 5 + 15 = 20 6. If × stands for 'addition', ÷ stands for 'subtraction', + stands for 'multiplication' and-stands for 'division', then 20 × 8 ÷ 8 - 4 + 2 = ? A. 80 B. 25 C. 24 D. 5 Answer: C. 24

116

Logical Reasoning

Nithra

Explanation: By the Given data, We have the expression: 20 + 8 - 8 ÷ 4 × 2 = 20 + 8 - 2 × 2 = 20 + 8 - 4 = 24. 7. If 2 + 3 = 10 8 + 4 = 96 7 + 2 = 63 6 + 5 = 66 then 9 + 5 = ? A. 99 B. 89 C. 116 D. 126 Answer: D. 126 Explanation: Add the given number and multiply the first number 2 + 3 = 5 × 2 = 10 8 + 4 = 12 × 8 = 96 7 + 2 = 9 × 7 = 63 6 + 5 = 11 × 6 = 66 9 + 5 = 14 × 9 = 126.

117

Logical Reasoning

Nithra

8. If 2 + 2 = 6 3 + 3 = 11 4 + 4 = 18 then 6 + 6 = ? Answer : 38 Explanation : Multiply by the given number and add 2 (2 * 2) + 2 = 6 (3 * 3) + 2 = 11 (4 * 4) + 2 = 18 (6 * 6) +2 = 38 9. If 2 + 3 = 10 8 + 4 = 96 7 + 2 = 63 6 +5 = 66 then 9 + 5 = ? Answer : 126 Explanation : Add the given number and multiply the first number 2 +3 = 5 * 2 = 10 8 + 4 = 12 * 8 = 96 7 + 2 = 9 * 7 = 63 6 + 5 = 11 * 6 = 66 9 + 5 = 14 * 9 = 126

118

Logical Reasoning

Nithra

10. If 1 = 17 2 = 33 3 = 49 4 = 65 then 5 = ? Answer : 81 Explanation : Multiply by 16 and add 1 (1 * 16) + 1 = 17 (2 * 16) + 1 = 33 (3 * 16) + 1 = 49 (4 * 16) + 1 = 65 (5 * 16) + 1 = 81.

Questions for Practice 1. If 2 + 3 = 13 3 + 4 = 25 4 +5 = 41 then 5+6=? 2 . If 3, 2 = 13; 4, 8 = 80; then 1, 5 =? 3. If 1 + 5 = 12 2 + 10 = 24 3 + 15 = 36 then 3 + 25 = ?

119

Logical Reasoning

Nithra

4. Find out the two signs to be interchanged for making following equation correct: 5 + 3 * 8 - 12 / 4 =3 5. Find out the two signs to be interchanged for making following equation correct: 5 + 3 * 8 - 12 / 4 = 3

Blood Relation Tricks Father’s side – Paternal Mother’s side – Maternal Mother’s or Father’s Father – Grandfather (Maternal or Paternal) Mother’s or Father’s Mother – Grandmother (Maternal or Paternal) Mother’s or Father’s Brother – Uncle Mother’s or Father’s Sister – Aunt Mother’s or Father’s Son – Brother Mother’s or Father’s Daughter – Sister Son’s Wife – Daughter-in-law Daughter’s Husband – Son-in-law Husband’s or wife’s Father – Father-in-law Husband’s or wife’s Mother – Mother-in-law Husband’s or wife’s Brother – Brother-in-law Husband’s or wife’s Sister – Sister-in-law Sister’s Husband – Brother-in-law Brother’s Wife – Sister-in-law Brother’s or Sister’s Son – Nephew Brother’s or Sister’s Daughter – Niece Uncle’s or Aunt’s Son or Daughter – Cousin (He is considered as First Cousin) Son or Daughter of the First Cousin – Cousin (He is mainly addressed as Second Cousin) Grandson’s or Grand Daughter’s Son – Great Grand son Grandson’s or Grand Daughter’s Daughter – Great Grand Daughter Father’s or Mother’s Aunt – Grand Aunt or Grand Aunt 120

Logical Reasoning

Nithra

Father’s or Mother’s Uncle Real Brother or Sister (Common Parents)

– Grand Uncle or Great Uncle – Sibling

Solved Problems Directions (1-3): Study the following information carefully to answer the questions that follow: 1. A is father of B and C is mother of A. E is sister of F whose daughter is G. S, the husband of C is the grandfather of G. P is father of E and brother of R. S has only two children, both of opposite sex. 1. What is the relation between F and S? A. F is daughter of S B. F is sister of S C. F is son of S D. F is daughter in law of S Answer: D. F is daughter in law of S Explanation: C and S have 2 children, one is A who is male [A is father of B]. since C is the grandfather of G so F should be wife of A to have the relation C is the grandfather of G. If F was daughter of C and s, so C would have been maternal grandfather of her daughter G.

121

Logical Reasoning

Nithra

2. What is the relation between E and B? A. E is sister of B B. E is brother of B C. E is aunt of B D. E is maternal grandmother of B Answer: Answer: C. E is aunt of B. Explanation: E is sister of F and B is child of F. 3. What is the relation between B and G? A. B is sister of G B. B is brother of G C. B is aunt of G D. None of these Answer: D. None of these Explanation: Since gender of B is not known, either sister or brother of G

122

Logical Reasoning

Nithra

4. Introducing a man, a woman said, "He is the only son of the mother of my mother." How is the woman related to the man? A. Mother B. Niece C. Sister D. Maternal aunt Answer: B. Niece Explanation : The man is the only son of the mother of the woman. Hence, the man is the maternal uncle of the woman. So, the woman is the niece of the man 5. Pointing out to a lady, Yuvarani said, she is the daughter of the woman who is the mother of the husband of my mother. How is the lady related to Yuvarani? A. Aunt B. Grand daughter C. Sister D. Daughter Answer: A. Aunt Explanation : Mother's husband = Father, Father's mother = Grandmother, Grandmother's daughter = Father's sister, Father's sister = Aunt So, the lady is Yuvarani's aunt.

123

Logical Reasoning

Nithra

6. A man said to a lady, Your mother's husband's sister is my aunt. How is the lady related to the man? A. Daughter B. Mother C. Sister D. Aunt Answer: C. Sister Explanation : Lady's mother's husband = lady's father Lady's father's sister = lady's aunt So, lady's aunt is man's aunt and Therefore the lady is man's sister. 7. Pointing to a photograph, Sathis said, "She is the daughter of my grandfather's only son". How is Sathis related to the girl in the photograph ? A.) Father B.) Brother C.) Uncle D.) Cousin Answer: B.) Brother Explanation : ✓ My grandfather's only son - My father. ✓ So, the girl is the daughter of Sathis's father. ✓ Sathis is the girl's brother.

124

Logical Reasoning

Nithra

8. Looking at a portrait of a man, Santhosh said, "His mother is the wife of my father's son. Brothers and sisters I have none." At whose portrait was Santhosh Looking. A. His uncle B. His nephew C. His cousin D. His son Answer: D. His son Explanation : ✓ Since, Santhosh has neither a sister nor a brother, therefore, Santhosh is the only son of his father. ✓ Hence, the mother of the portrait is wife of Santhosh. ✓ Therefore, the portrait is the wife of the Santhosh. ✓ Therefore, portrait was of Santhosh's son. 9. Ramesh said to Sada, "Your mother is the daughter of my grandmother". How are Ramesh and Sada related ? A. Cousin B. Father - Daughter C. Uncle - Niece D. None of these Answer: A. Cousin Explanation : ✓ Grandmother - mother ✓ Sada's mother - Daughter ✓ Ramesh - Sada Her Cousin

125

Logical Reasoning

Nithra

10. X and Y are young ones of Z. If Z is the father of X but Y is not the son of Z. How are Y and Z related A. Newphew and Uncle B. Niece and Uncle C. Daughter and Father D. None of these Answer: C. Daughter and Father Explanation : Z has two children X and Y, if X is his son, Y must be his daughter according to the question.

Questions for Practice 1. Pointing towards a person in a photograph, Monisha said, "Surya is the only son of the father of my sister's brother." How is that person is related to Monisha ? A) Monisha Father B) Monisha Mother C) Monisha Brother D) Maternal Uncle 2. Anupriya told Sumitha,'Yesterday I defeated the only brother of the daughter of my grandmother'. Whom did Anupriya defeat? A. Father-in-law B. Father C. Brother D. Son 126

Logical Reasoning

Nithra

3.If Raja says, "Masi's mother is the only daughter of my mother", how is Raja related to Masi ? A) Brother B) Father C) Maternal Uncle D) Cousin

4. A is B's sister. C is B's mother. D is C's father. E is D's mother. Then, how is A related to D? A) Grandfather B) Grandmother C) Daughter D) Grand daughter

5. If P $ Q means P is the brother of Q P # Q means P is the mother of Q P * Q means P is the daughter of Q in A # B $ C * D, who is the father? A) A. D B) B. B C) C.C D) Data is inadequate

Cause and Effect Solved Problems In each of the following questions, two statements numbered I and II are given. There may be cause and effect relationship between the two statements. These two statements may be the effect of the same cause or independent causes. These statements may be independent causes without having any relationship. Read both the statements in each question and mark your answer as 127

Logical Reasoning

Nithra

1. Statements: I) The prices of petrol and diesel in the domestic market have remained unchanged for the past few months. II ) The crude oil prices in the international market have gone up substantially in the last few months. A. Statement I is the cause and statement I is its effect B. Statement II is the cause and statement I is its effect C. Both the statements I and II are independent causes D. Both the statements I and II are effects of independent causes E. Both the statements I and II are effects of some common cause Answer : D. Both the statements I and II are effects of independent causes Explanation : The prices of petrol and diesel being stagnant in the domestic market and the increase in the same in the international market must be backed by independent causes. 2. Statements: I) The government has recently fixed the fees for professional courses offered by the unaided institutions which are much lower than the fees charged last year. II) The parents of the aspiring students launched a severe agitation last year protesting against the high fees charged by the unaided institutions. A. B. C. D.

Statement I is the cause and statement II is its effect Statement II is the cause and statement I is its effect Both the statements I and II are independent causes Both the statements I and II are effects of independent causes

Answer : B. Statement II is the cause and statement I is its effect

128

Logical Reasoning

Nithra

Explanation : The parents protest against high fees being charged by the institutions led the government to interfere and fix the fees at a more affordable level. 3. Statements: I. The government has recently fixed the fees for professional courses offered by the unaided institutions which are much lower than the fees charged last year. II.The parents of the aspiring students launched a severe agitation last year protesting against the high fees charged by the unaided institutions. A. Statement I is the cause and statement II is its effect B. Statement II is the cause and statement I is its effect C. Both the statements I and II are independent causes D. Both the statements I and II are effects of independent causes E. Both the statements I and II are effects of some common cause Answer : B. Statement II is the cause and statement I is its effect Explanation : The parents protest against high fees being charged by the institutions led the government to interfere and fix the fees at a more affordable level. 4. Statements: i. All the schools in the area had to be kept closed for most part of the week. ii. Many parents have withdrawn their children from the local schools. A. Statement I is the cause and statement II is its effect B. Statement II is the cause and statement I is its effect C. Both the statements I and II are independent causes D. Both the statements I and II are effects of independent causes

129

Logical Reasoning

Nithra

Answer : D. Both the statements I and II are effects of independent causes Explanation : Closing the schools for a week and the parents withdrawing their wards from the local schools are independent issues, which must have been triggered by different individual causes. 5. Statements: I. India has surpassed the value of tea exports this year over all the earlier years due to an increase in demand for quality tea in the European market. II. There is an increase in demand of coffee in the domestic market during the last two years. A. Statement I is the cause and statement II is its effect B. Statement II is the cause and statement I is its effect C. Both the statements I and II are independent causes D. Both the statements I and II are effects of independent causes E. Both the statements I and II are effects of some common cause Answer : C. Both the statements I and II are independent causes Explanation : The two statements discuss two separate statistical and generalised results.

130

Logical Reasoning

Nithra

6. Statements: i. There is unprecedented increase in the number of young unemployed in comparison to the previous year. ii. A large number of candidates submitted applications against an advertisement for the post of manager issued by a bank. A. Statement I is the cause and statement II is its effect B. Statement II is the cause and statement I is its effect C. Both the statements I and II are independent causes D. Both the statements I and II are effects of independent causes Answer : A. Statement I is the cause and statement II is its effect Explanation : An increase in the number of unemployed youths is bound to draw in huge crowds for a single vacancy. 7. Statements: (A) The shortage of employable talent has become severe. (B) There are more job opportunities than there are qualified professionals. A. Statement I is the cause and statement II is its effect B. Statement II is the cause and statement I is its effect C. Both the statements I and II are independent causes D. Both the statements I and II are effects of independent causes Answer : A. if statement (A) is the cause and statement (B) is its effect. Explanation : As the number of talented people is less, those with talent are getting more opportunities. Hence, (A) is the cause and (B) is its effect.

131

Logical Reasoning

Nithra

8. Statements: I. Majority of the students in the college expressed their opinion against the college authority's decision to break away from the university and become autonomous. II. The university authorities have expressed their inability to provide grants to its constituent colleges. A. Statement I is the cause and statement II is its effect B. Statement II is the cause and statement I is its effect C. Both the statements I and II are independent causes D. Both the statements I and II are effects of independent causes Answer : B. Statement II is the cause and statement I is its effect Explanation : Clearly, the university's decision to refuse grant to the colleges must have triggered the college authority to become autonomous. 9. Statements: (A) The destiny of Asia will be shaped by the triangular relationship between the Asian nations, India, China and Japan. (B) India, China and Japan which have registered a phenomenal growth, over the last decade in particular, has acted as southern engines of growth for the whole world. A. if statement (A) is the cause and statement (B) is its effect. B. if statement (B) is the cause and statement (A) is its effect. C. if both the statements (A) and (B) are independent causes. D. if both the statements (A) and (B) are effects of independent causes. Answer : B. if statement (B) is the cause and statement (A) is its effect.

132

Logical Reasoning

Nithra

Explanation : Since the three countries are leading the economic growth of the whole world, they will shape the destiny of Asia. Hence, (B) is the cause and (A) is its effect. 10. Statements: I) Shinzo Abi has resigned as the Prime Minister of Japan. II) This is an ignominious exit for Shinzo Abi who was once received as the standard bearer of the new generation of Japanese politicians. A. if both the statements are effect of some common causes. B. if statement (B) is the cause and statement (A) is its effect. C. if both the statements (A) and (B) are independent causes. D. if both the statements (A) and (B) are effects of independent causes. Answer : A. if both the statements are effect of some common causes. Explanation : The reason for the exit being ignominious and his resignation are one and the same. Hence, (A) and (B) are effects of a common cause.

Questions for Practice In each of the following questions, two statements numbered I and II are given. There may be cause and effect relationship between the two statements. These two statements may be the effect of the same cause or independent causes. These statements may be independent causes without having any relationship. Read both the statements in each question and choose your answer.

133

Logical Reasoning

Nithra

1. Statement: (A) The single child family seems to be fast becoming the norm in the present-day urban set up. (B) People think that parenting is easier when they have only one child. A. if statement (A) is the cause and statement (B) is its effect. B. if statement (B) is the cause and statement (A) is its effect. C. if both the statements (A) and (B) are independent causes. D. if both the statements (A) and (B) are effects of independent causes. E. if both the statements are effect of some common causes. 2. Statement: (A) English-medium schools and institutes teaching spoken English, are mushrooming even in small towns. (B) Today, more than ever before, there is a craving among people to learn and speak English. A. if statement (A) is the cause and statement (B) is its effect. B. if statement (B) is the cause and statement (A) is its effect. C. if both the statements (A) and (B) are independent causes. D. if both the statements (A) and (B) are effects of independent causes. E. if both the statements are effect of some common causes. 3. Statement: (A) The shortage of employable talent has become severe. (B) There are more job opportunities than there are qualified professionals. A. if statement (A) is the cause and statement (B) is its effect. B. if statement (B) is the cause and statement (A) is its effect. C. if both the statements (A) and (B) are independent causes. D. if both the statements (A) and (B) are effects of independent causes. E. if both the statements are effect of some common causes.

134

Logical Reasoning

Nithra

In each of the following questions, two statements numbered I and II are given. There may be cause and effect relationship between the two statements. These two statements may be the effect of the same cause or independent causes. These statements may be independent causes without having any relationship. Read both the statements in each question and choose your answer. 4. Statement: (A) Importance of Yoga and exercise is being realized by all sections of the society. (B) There is an increasing awareness about health in the society particularly among middle ages group of people. A. B. C. D.

Statement I is the cause and statement II is its effect. Both the statements I and II are effects of some common cause. Statement II is the cause and statement I is its effect. Both the statements I and II are effects of independent causes.

Answer : C. Statement II is the cause and statement I is its effect. 5) Statement: (A) Indonesia suffered a major earth quake today. (B) The term of the Indonesian prime Minister ends by December. A. if statement (B) is the cause and statement (A) is its effect. B. if both the statements (A) and (B) are independent causes. C. if statement (A) is the cause and statement (B) is its effect. D. if both the statements (A) and (B) are effects of independent causes.

Data Sufficiency Data sufficiency covers many different topics of quantitative aptitude. In data sufficiency, usually, a question is followed by two or three statements. You need to determine whether any of the statements individually or together are required to find the answer. You are not required to do the calculation; you just have to check whether with the help of given data you can find the answer or not. Data sufficiency 135

Logical Reasoning

Nithra

has many types of questions. And today we will be discussing on CI and SI-based data sufficiency. The formula to calculate simple interest I = P x R x N/100, where p is the principal amount, r is the rate of interest, and n is the number of years Here, A is the total amount i.e. principal + interest, P is the principal amount, R is the rate of interest, and N is the number of years. There are other two formulas as well to calculate compound interest quarterly and half-yearly. Compound interest for quarterly,A = P (1 + R/100 x 2)²n Compound interest for half-yearly,A = P (1 + R/100 x 4)4n

Solved Problems 1. What is Reena's rank in the class ? I. There are 26 students in the class. II. There are 9 students who have scored less then Reena. A. I alone is sufficient while II alone is not sufficient B. II alone is sufficient while I alone is not sufficient C. Either I or II is sufficient D. Neither I nor II is sufficient E. Both I and II are sufficient. Answer: Option E Explanation: From I and II, we conclude that there are 16 students above Reena in rank. Thus, Reena's rank is 17th in the class. So, both the statements are necessary.

136

Logical Reasoning

Nithra

2. Who is the father of M? I. A and B are brothers. II. B's wife is sister of M's wife. A. if the data in statement I alone are sufficient to answer the question; B. if the data in statement II alone are sufficient to answer the question; C. if the data either in I or II alone are sufficient to answer the question; D. if the data even in both the statement together are not sufficient to answer the question; E. if the data in both the statements together are needed. Answer: Option D : Neither I nor II is sufficient. Explanation: From II, we conclude that B is the brother-in-law of M. So, even from both the statements, we can't find out who is the father of M. 3. In which year was Rohit born ? I. Rohit at present is 25 years younger to his mother. II. Rohit's brother, who was born in 1964, is 35 years younger to his mother. A. If the data in statement I alone are sufficient to answer the question B. If the data in both the statements together are needed. C. If the data either in I or II alone are sufficient to answer the question D. If the data in statement II alone are sufficient answer the question Answer : B. If the data in both the statements together are needed. Explanation : From both the given statements, we find that Rohit is (35 - 25) = 10 years older than his brother who was born in 1964, So. Rohit was horn in 1954. 137

Logical Reasoning

Nithra

Thus, both the given statements are needed to answer the query. 4. How is P related to Q ? I.P has only one son and two daughters II.Q's brother is son of P's wife . A. If the data in statement I alone are sufficient B. If the data given in both I and II together are not sufficient C. If the data either in statement I alone or statement II alone are sufficient to answer D. If the data in statement II alone are sufficient Answer : D. If the data in statement II alone are sufficient Explanation : From II, we know that P is father of Q 5. What is the shortest distance between Devipur and Durgapur ? I. Durgapur is 20 kms away from Rampur. II. Devipur is 15 kms away from Rampur. A. I alone is sufficient while II alone is not sufficient B. II alone is sufficient while I alone is not sufficient C. Neither I nor II is sufficient D. Either I or II is sufficient Answer : C. Neither I nor II is sufficient Explanation : Clearly, the distance of each village from Rampur is given in I and II. But nothing about their relative positions is mentioned. So, the distance between the two villages cannot be calculated.

138

Logical Reasoning

Nithra

6. On which day in April is Ganesh's birthday? I. Ganesh was born exactly 28 years after his mother was born. II. His mother will be 55 years 4 months and 5 days on August 18 this year. A. I alone is sufficient while II alone is not sufficient B. II alone is sufficient while I alone is not sufficient C. Either I or II is sufficient D. Both I and II are sufficient Answer : D. Both I and II are sufficient Explanation : Clearly, the birthday of Ganesh's mother can be found out from II and then Ganesh's birthday can be determined using the fact given in I. 7. How much amount Ravi required to pay for the new car in the buy-back scheme ? I. The cost of the new car was three times the cost price of his old car. II. His old car was valued at Its 25000 under buy-back scheme. A. If the data in statement I alone are sufficient to answer the questio B. If the data in statement II alone are sufficient answer the question C. If the data in both the statements together are needed. D. If the data either in I or II alone are sufficient to answer the question Answer : C. If the data in both the statements together are needed. Explanation : From both the statements together, we conclude that the price of the new car is Rs (25000 X 3) i.e. Rs 75000 So, Ravi had to pay Rs (75000 - 25000) i.e. Its 50000. Thus, both I and II are required

139

Logical Reasoning

Nithra

8. In a certain language, 'pit nac mit' means 'red pant shirt'. Which word means 'pant' in that language? I. 'mit tim nac sir' means 'he wore red pant'. II. 'nee jic pit' means 'shirt is dirty'. A. if the data in statement I alone are sufficient to answer the question B. if the data in statement II alone are sufficient answer the question C. if the data either in I or II alone are sufficient to answer the question D. if the data even in both the statements together are not sufficient to answer the question Answer : D. if the data even in both the statements together are not sufficient to answer the question Explanation : Clearly, from each of the statements, we find that the code for 'pant' is either 'mit' or 'nac'. So, none of them is sufficient to answer the question. 9.Is Arun taller than Sachin ? I. Dinesh is of the same height as Arun and Sachin. II. Sachin is not shorter than Dinesh. A. if the data in statement I alone are sufficient to answer the question; B. if the data in statement II alone are sufficient answer the question; C. if the data either in I or II alone are sufficient to answer the question; D. if the data even in both the statements together are not sufficient to answer the question; E. if the data in both the statements together are needed. Answer: Option A : if the data in statement I alone are sufficient to answer the question

140

Logical Reasoning

Nithra

Explanation: From statement I, we can conclude that Dinesh, Arun and Sachin are of the same height. So, Arun is not taller than Sachin. Thus, only statement I is sufficient to answer the question. 10. What is the monthly salary of Praveen? I. Praveen gets 15% more than Sumit while Sumit gets 10% less than Lokesh. II. Lokesh's monthly salary is Rs.2500. A. if the data in statement I alone are sufficient to answer the question B. if the data in statement II alone are sufficient answer the question C. if the data either in I or II alone are sufficient to answer the question D. if the data even in both the statements together are not sufficient to answer the question E. If the data in both the statements together are needed. Answer : E. If the data in both the statements together are needed. Explanation : From both the given statements, we find that: Praveen's salary = 115% of (90% of Rs.2500) = Rs.2587.50. So, both I and II are required.

Questions for Practice 1. How is D related to A? A. if the data in statement I alone are sufficient to answer the question; B. if the data in statement II alone are sufficient answer the question; C. if the data either in I or II alone are sufficient to answer the question; D. if the data even in both the statements together are not sufficient to answer the question; E. If the data in both the statements together are needed. 141

Logical Reasoning

Nithra

2. Gourav ranks eighteenth in a class. What is his rank from the last? I. There are 47 students in the class. II. Jatin who ranks 10th in the same class, ranks 38th from the last. A. if the data in statement I alone are sufficient to answer the question; B. if the data in statement II alone are sufficient answer the question; C. if the data either in I or II alone are sufficient to answer the question; D. if the data even in both the statements together are not sufficient to answer the question; E. If the data in both the statements together are needed. 3. How many brothers does Tarun have? A. if the data in statement I alone are sufficient to answer the question; B. if the data in statement II alone are sufficient answer the question; C. if the data either in I or II alone are sufficient to answer the question; D. if the data even in both the statements together are not sufficient to answer the question; E. If the data in both the statements together are needed. 4.Rohit, Kajol, Tanmay and Suman are four friends. Who is the oldest among them? I. The total age of Kajol and Tanmay together is more than that of Suman. II. The total age of Rohit and Kajol together is less than that of Suman. A. if the data in statement I alone are sufficient to answer the question; B. if the data in statement II alone are sufficient answer the question; C. if the data either in I or II alone are sufficient to answer the question; D. if the data even in both the statements together are not sufficient to answer the question; E. If the data in both the statements together are needed.

142

Logical Reasoning

Nithra

5. How many new year's greeting cards were sold this year in your shop? I. Last year 2935 cards were sold. II. The number of cards sold this year was 1.2 times that of last year. A. if the data in statement I alone are sufficient to answer the question; B. if the data in statement II alone are sufficient answer the question; C. if the data either in I or II alone are sufficient to answer the question; D. if the data even in both the statements together are not sufficient to answer the question; E. If the data in both the statements together are needed.

Direction Sense Test Solved Problems 1. One morning after sunrise Naveena and Nirajana were talking to each other face to face at Dalphin crossing. If Nirajana's shadow was exactly to the right of Naveena, Which direction Nirajana was facing? A. East B. South C. North D. Data is inadequate Answer : C. North Explanation : In the morning sun rises in the East. Hence then any shadow falls in the West. Since Nirajana's shadow was exactly to the right of Naveena. Hence Nirajana is facing towards North.

143

Logical Reasoning

Nithra

2. One evening before sunset two friends Sachin and Dhoni were talking to each other face to face. If Dhoni's shadow was exactly to his right side, which direction was Sachin facing? A. North B. South C. West D. Data inadequate Answer : B. South Explanation : In the evening, sun is in the west and so the shadows fall towards east. Now, since Dhoni's shadow fell towards right, therefore, Dhoni is facing North. So, Sachin standing face to face with Dhoni, was facing South. 3. Kholi starts from his house towards West. After walking a distance of 30 meters, he turned towards right and walked 20 meters. He turned left and moving a distance of 10 meters, turned to his left again and walked 40 meters. He now turns to the left and walks 5 meters. Finally he turns to his left. In which direction is he walking now? A. East B. South C. North D. South-west Answer : C. North

144

Logical Reasoning

Nithra

4. One morning after sunrise, Kumar was standing facing a pole. The shadow of the pole fell exactly to his right. Which direction was he facing? A. Data inadequate B. East C. West D. South Answer : D. South Explanation : Sun rises in the east in the morning. So, in morning, the shadow falls towards west. Now Kumar's shadow falls to the right. So, he is standing, facing South. 5. Fifteen boys are standing in a row facing opposite direction alternately from left to right. If the fourth boy from left is facing towards the east then the fifth boy from the right is facing which direction? A. South B. North-west C. None of these D. East Answer : C. None of these Explanation : Fifth boy from right means (15-5)+1= 11th boy from left.

145

Logical Reasoning

Nithra

6. Deepak starts walking straight towards east. After walking 75 meters, he turns to the left and walks 25 meters straight. Again he turns to the left, walks a distance of 40 meters straight, again he turns to the left and walks a distance of 25 meters. How far is he from the starting point? A. 25 meters B. 50 meters C. 115 meters D. 140 meters E. None of these Answer : E. None of these Explanation :

The movements of Deepak are shown in the figure. Clearly, EB = DC = 40 m. Deepak's distance from the starting point A = (AB - EB) = (75 - 40)m = 35 m.

146

Logical Reasoning

Nithra

7. One morning after sunrise Rathipriya and Latha were talking to each other face to face at Dalphin crossing. If Latha's shadow was exactly to the right of Rathipriya, Which direction Latha was facing? A. South B. North C. East D. Data is inadequate Answer : B. North Explanation : In the morning sun rises in the East. Hence then any shadow falls in the West. Since Latha's shadow was exactly to the right of Rathipriya. Hence Latha is facing towards North. 8. A man leaves for his office from his house. He walks towards East. After moving a distance of 20 m, he turns towards South and walks 10 m. Then he walks 35 m towards the West and further 5 m towards the North. He then turns towards East and walks 15 m. What is the straight distance in meters between his initial and final positions? A. 0 B. 5 C. 10 D. Cannot be determined Answer : B. 5

147

Logical Reasoning

Nithra

Explanation :

The movements of the man from A to F are as shown in figure. Clearly, DC = AB + EF. F is in line with A. Also, AF = (BC - DE) = 5 m. So, the man is 5 meters away from his initial position. 9. One morning after sunrise, Suresh was standing facing a pole. The shadow of the pole fell exactly to his right. To which direction was he facing? A. East B. West C. South D. Data is inadequate Answer : C. South Explanation : Sun rises in the east in the morning. Since the shadow of Suresh falls to his right. So he is facing South.

148

Logical Reasoning

Nithra

10. Arun and Amit started walking from two different points 'A' and 'B' respectively. Arun walks 2 kms North and turns to the East and walks 3 kms and again turns to North walks 4 kms and finally turns to East and Walks 5kms to reach point 'C'. Similarly, Amit walks 2 kms North and turns to west and walks 3 kms and finally turns to North, walks 4 kms and meets Arun at point 'C'. What is the distance between Arun and Amit's starting points? Answer : 11 km Explanation :

Where A= Arun's starting point; B = Amit's starting point; C = The point where Arun and Amit meet PQ + RC + ST = 3 + 5 + 3 = 11 kms.

Questions for Practice 1. After walking 6 kms, I turned right and travelled a distance of 2 kms, then turned left and covered a distance of 10 km. In the end I was moving towards the north. From which direction did I start my journey? A. North B. South-West C. South D. North-East

149

Logical Reasoning

Nithra

2. One evening, two friends Prasanth and Parthipan were talking to each other, with their backs towards each other, sitting in a park. If Prasanth's shadow was exactly to the left of Parthipan, then which direction was Parthipan facing? A. North B. North-East C. East D. South 3. Fifteen boys are standing in a row facing opposite direction alternately from left to right. If the fourth boy from left is facing towards the east then the fifth boy from the right is facing which direction? A. South B. North-west C. None of these D. East 4. Raja left home and cycled 10 km towards South, then turned right and cycled 5 km and then again turned right and cycled 10 km. After this he turned left and cycled 10 km. How many kilometers will he have to cycle to reach his home straight? A. 10 km B. 25 km C. 20 km D. 15 km

150

Logical Reasoning

Nithra

5. One morning after sunrise Rathipriya and Latha were talking to each other face to face at Dalphin crossing. If Latha's shadow was exactly to the right of Rathipriya, Which direction Latha was facing? A. South B. North C. East D. Data is inadequate

Linear inequality Definition In mathematics a linear inequality is an inequality which involves a linear function. A linear

Solved Problems Directions (1-3): In the following questions, the symbols @, $, *, # and ! are used with the following meaning as illustrated below: P $ Q means P is not smaller than Q. P @ Q means P is neither smaller than nor equal to Q. P # Q means P is neither greater than nor equal to Q. P ! Q means P is neither greater than nor smaller than Q. P * Q means P is not greater than Q. 1. Statements: M $ K; K @ N; N * R; R # W. Conclusions: I. W @ K II. M $ R III. K @ W 151

Logical Reasoning

Nithra

IV. M @ N A. Only I and II follows B. Only I, II and III follows C. Only IV follows D. Only II, III and IV follow Answer: C. Only IV follows. Explanation: M ≥ K, K >N, N ≤ R, R <W W >K not follows M ≥ R not follows K >W not follows M >N follows 2. Statements: H @ T; T # F; F ! E; E * V. Conclusions: I. V $ F II. E @ T III. H @ V IV. T # V A. Only I, II and III follows B. Only I, II and IV follows C. Only II, III and IV follows D. All I, II, III and IV follows Answer: B. Only I, II and IV follows.

152

Logical Reasoning

Nithra

Explanation: H >T, T T follows H >V not follows T W H >N W=N W
153

Logical Reasoning

Nithra

Directions : In the following questions, the symbols $,@,%, © and # are used with the following meanings as illustrated below: A©B means A is greater than B. A%B means A is smaller than B. A@B means A is either greater than or equal B. A$B means A is either smaller than or equal to B. A#B means A is equal to B. 4. Statements: V $ Z; Z © M; M % H. Conclusions: I. M @ V II. H # V III. V © M A. Only I and II follows B. Only II and III follow C. Only I and III follow D. Either l or lll follows Answer: D. Either l or lll follows Explanation: V ≤ Z, Z >M, M M

154

Logical Reasoning

Nithra

Study the following information carefully and answer the questions given below: (i) A, B, C, D, E, F, G and H are eight students, each having a different height. (ii) D is shorter than A but taller than G (iii) E is taller than H but shorter than C (iv) B is shorter than D but taller than F (v) C is shorter than G (vi) G is not as tall as F 5. Which of the following is definitely false? A.G is shorter than F B. C is shorter than F C. F is taller than c D. B is taller than E E. All are true Answer: E. All are true Explanation: A>D>G C>E>H D>B>F G>C F>G Combining these , we get A > D > B > F > G > C > E > H is the correct answer. 6. If another student J, who is taller than E but shorter than G, is added to the group, which of the following will be definitely true? A. C and J are of the same height B. J is shorter than D C. J is shorter than H D. J is taller than A E.None of these 155

Logical Reasoning

Nithra

Answer: B. J is shorter than D Explanation: A>D>G C>E>H D>B>F G>C F>G Combining these , we get A > D > B > F > G > C > E > H is the correct answer; A > D > B > F > G > C, J > E > H 7. Which of the following will definitely be the third from top when the eight student are arranged in descending order of height? A. B B. F C. G D. B or G E. Cannot be detemined Answer: A. B Explanation: A>D>G C>E>H D>B>F G>C F>G Combining these, we get A > D > B > F > G > C > E > H is the correct answer.

156

Logical Reasoning

Nithra

8. How many of them are definitely shorter than F? A. Three B. Four C. Five D. Data inadequate E. None of these Answer: B. Four Explanation: A>D>G C>E>H D>B>F G>C F>G Combining these , we get A > D > B > F > G > C > E > H is the correct answer; G, C, E and H. 9. Which of the following is redundant to answer all the above questions? A. (ii) only B. (ii) and (iii) only C. (iii) and (iv) only D. (ii) and (v) only E. All are necessary to answer the above questions Answer: E. All are necessary to answer the above questions Explanations : A>D>G C>E>H D>B>F G>C 157

Logical Reasoning

Nithra

F>G Combining these , we get A > D > B > F > G > C > E > H is the correct answer. 10. Considering all the statement in each question true, find that which of the following two conclusion – I & II is definitely true. Give the answer (A) if only conclusion I is true. Give the answer (B) if only conclusion II is true. Give the answer (C) if either conclusion I or conclusion II is true. Give the answer (D) if neither conclusion I nor conclusion II is true. Give the answer (E) if conclusion I & conclusion II – both are true. 1. Statement: L * P, P % V, V # D Conclusion: I. L * V II. L $ D Answer : D. if neither conclusion I nor conclusion II is true.

Questions for Practice Direction: (1-5) : In these question symbols $, #, %, © & @ are used for different meaning as follows: ‘A $ B’ means A is less than B. ‘A # B’ means A is not less than B. ‘A % B’ means A is neither less than nor greater than B. ‘A © B’ means A is greater than B. ‘A @ B’ means A is not greater than B. Considering all the statement in each question true, find that which of the following two conclusions – I & II is definitely true. Give the Answer – (A) If only conclusion I is true. (B) If only conclusion II is true. (C) If neither conclusion I or conclusion II is true. 158

Logical Reasoning

Nithra

(D) If neither conclusion I nor conclusion II is true. (E) If conclusion I & conclusion II – both are true. 1. Statement: D $ L, L @ V, V # W Conclusion: I. D $ V II. D @ W 2. Statement: M # T, T © H, H @ F Conclusion: I. M © H II. T © F 3. Statement: H © L, L % E, E $ T Conclusion: I. H @ T II. H © T 4. Statement: L @ R, R % J, J © N Conclusion: I. L @ N II. N $ R 5. Statement: V # F, F © J, J % D Conclusion: I. V # D II. F © D

Logical Deductions Definition The phenomenon of deriving a conclusion from a single proposition or a set of given propositions, is known as logical deduction. The given propositions are also referred to as the premises. Two Inferential Processes of Deduction I. Immediate Deductive Inference : II. Mediate Deductive Inference 159

Logical Reasoning

Nithra

I. Immediate Deductive Inference : Here, conclusion is deduced from one of the given propositions, by any of the three ways -conversion, obversion and contraposition. 1. Conversion: The Conversion proceeds with interchanging the subject term and the predicate term i.e. the subject term of the premise becomes the predicate term of the conclusion and the predicate term of the premise becomes the subject of the conclusion. The given proposition is called convertend, whereas the conclusion drawn from it is called its converse. Table of Valid Conversions Converted

Converse

A: All S is Ex. All pins are tops.

P I: Some P is S Some tops are pins.

E: No S is Ex. No fish is whale.

P. E: No P is No whale is fish.

S.

I: Some P is S. I: Some S is P. Some poets are Ex. Some boys are poets. boys. O: Some S is not P.

No valid conversion

Note that in a conversion, the quality remains the same and the quantity may change. 2. Obversion: In obversion, we change the quality of the proposition and replace the predicate term by its complement.

160

Logical Reasoning

Nithra

Table of Valid Obversions Obverted

Obverse

A: All birds are mammals.

E: No birds are non-mammals.

E: No poets are singers.

A: All poets are non-singers.

I: Some nurses are doctors.

O: Some nurses are not non-doctors.

O: some statesmen.

politicians

are

not I: Some politicians statesmen.

are

non-

3. Contra position: To obtain the contra positive of a statement, we first replace the subject and predicate terms in the proposition and then exchange both these terms with their complements. Table of Valid Contra positions Proposition

Contra positive

A: All birds are mammals.

A: All non-mammals are non-birds.

I: Some birds are mammals.

I: Some non-mammals are nonbirds.

Note: The valid converse, obverse or contrapositive of a given proposition always logically follows from the proposition. II. Mediate Deductive Inference (SYLLOGISM): First introduced by Aristotle, a Syllogism is a deductive argument in which conclusion has to be drawn from two propositions referred to as the premises.

161

Logical Reasoning

Nithra

Example: 1. All lotus are flowers. 2. All flowers are beautiful. 3. All lotus are beautiful. Clearly, the propositions 1 and 2 are the premises and the proposition 3, which follows from the first two propositions, is called the conclusion. Term : In Logic, a term is a word or a combination of words, which by itself can be used as a subject or predicate of a proposition. Syllogism is concerned with three terms : 1. Major Term : It is the predicate of the conclusion and is denoted by P (first letter of 'Predicate'). 2. Minor Term: It is the subject of the conclusion and is denoted by S (first letter of 'Subject'). 3. Middle Term: It is the term common to both the premises and is denoted by M (first letter of 'Middle'). Example: Premises: 1. All dogs are animals. 2. All tigers are dogs. Conclusion : ✓ ✓ ✓ ✓

All tigers are animals. Here 'animals' is the predicate of the conclusion and so,.it is the major term. P. 'Tigers' is the subject of the conclusion and so, it is the minor term, S. 'Dogs' is the term common to both the premises and so, it is the middle term, M. ✓ Major And Minor Premises : Of the two premises, the major premise is that in which the middle term is the subject and the minor premise is that in which the middle term is the predicate. 162

Logical Reasoning

Nithra

Solved Problems 1. Statement: All men are dogs. All dogs are cats. Conclusions: I. All men are cats. II. All cats are men. A) If only conclusion I follows B) If only conclusion II follows C) If either I or II follows D) If neither I nor II follows E) If both I and II follow. Answer : B. If only conclusion II follows Explanation : Since both the premises are universal and one premise is negative, the conclusion must be universal negative and should not contain the middle term. So, only II follows. 2. Statement: Should a total ban be put on traping wild animals? Conclusions: I. Yes. Trappers are making a lot of money II. No. Bans on hunting and trapping are not effective A) Only 1 is true B) Only 2 is true C) Either 1 or 2 is true D) Neither 1 or 2 is true Answer : D) Neither 1 or 2 is true

163

Logical Reasoning

Nithra

Explanation : Ban is necessary to protect our natural environment. So none of the argument is strong enough. 3. Statement: All trucks fly. Some scooters fly. Conclusions: I. All trucks are scooters. II. Some scooters do not fly. A) Only conclusion I follows B) Only conclusion II follows C) Either I or II follows D) Both I and II follow Answer : D) Both I and II follow Explanation : Since the middle term 'fly' is not distributed even once in the premises, no definite conclusion follows. 4. Statement: Most teachers are boys. Some boys are students. Conclusions: I. Some students are boys. II. Some teachers are students. A) Only conclusion I follows B) Only conclusion II follows C) Either I or II follows D) Both I and II follow Answer : A) Only conclusion I follows 164

Logical Reasoning

Nithra

Explanation : Since both the premises are particular, no definite conclusion follows. However, I is the converse of the second premise and thus it holds. 5. Statement: All men are dogs. All dogs are cats. Conclusions: I. All men are cats. II. All cats are men. A) If only conclusion I follows B) If only conclusion II follows C) If either I or II follows D) If neither I nor II follows E) If both I and II follow. Answer : B. If only conclusion II follows Explanation : Since both the premises are universal and one premise is negative, the conclusion must be universal negative and should not contain the middle term. So, only II follows. 6. Statement: Should a total ban be put on traping wild animals? Conclusions: I. Yes. Trappers are making a lot of money II. No. Bans on hunting and trapping are not effective A) Only 1 is true B) Only 2 is true C) Either 1 or 2 is true D) Neither 1 or 2 is true 165

Logical Reasoning

Nithra

Answer : D) Neither 1 or 2 is true Explanation : Ban is necessary to protect our natural environment. So none of the argument is strong enough 7. Statement: All trucks fly. Some scooters fly. Conclusions: I. All trucks are scooters. II. Some scooters do not fly. A) Only conclusion I follows B) Only conclusion II follows C) Either I or II follows D) Both I and II follow Answer : D) Both I and II follow Explanation : Since the middle term 'fly' is not distributed even once in the premises, no definite conclusion follows. 8. Statement: Most teachers are boys. Some boys are students. Conclusions: I. Some students are boys. II. Some teachers are students. A) Only conclusion I follows B) Only conclusion II follows C) Either I or II follows D) Both I and II follow 166

Logical Reasoning

Nithra

Answer : A) Only conclusion I follows Explanation : Since both the premises are particular, no definite conclusion follows. However, I is the converse of the second premise and thus it holds. 9. Statement: Raman is always successful. No fool is always successful. Conclusions: I. Raman is a fool II. Raman is not a fool A. Only I follows B. Only II follows C. Either I or II follows D. Both I and II follow Answer : B. Only II follows Explanation : Since both the premises are universal and one premise is negative, the conclusion must be universal negative and should not contain the middle term. So, II follows.

167

Logical Reasoning

Nithra

10. Statement: All trucks fly. Some Scooters fly. Conclusions: I.All trucks are scooters II. Some scooters do not fly A. Only I follows B. Only II follows C. Neither I nor II follows D. Both I and II follow Answer : C. Neither I nor II follows Explanation : Since the middle term "fly" is not distributed even once in the premises, so no definition conclusion follows.

Questions for Practice 1.Statement: All grasses are trees. No tree is shrub. Conclusions: I. No grasses are shrubs. II. Some shrubs are grasses. A. Only conclusion I follows B. Only conclusion II follows C. Either I or II follows D. Both I and II follow 2. Statement: All windows are doors. No door is wall Conclusions: I. No window is wall II. No Wall is door 168

Logical Reasoning

Nithra

A. Only I follows B. Only II follows C. Either I or II follows D. Both I and II follow 3. Statement: All snakes are trees. Some trees are roads. All roads are mountains. Conclusions: I. Some mountains are snakes II. Some roads are snakes III. Some mountains are trees A. Only I follows B. Only II follows C. Only III follows D. Both I and II follow 4. Statement: Every minister is a student. Every student is inexperienced. Conclusions: I. Every minister is inexperienced. II. Some inexperienced are students. A. Only conclusion I follows B. Only conclusion II follows C. Either I or II follows D. Both I and II follow

169

Logical Reasoning

Nithra

Making Judgments Solved Problems 1.The school principal has received complaints from parents about bullying in the school yard during recess. He wants to investigate and end this situation as soon as possible, so he has asked the recess aides to watch closely. Which situation should the recess aides report to the principal? A. A girl is sitting glumly on a bench reading a book and not interacting with her peers. B. Four girls are surrounding another girl and seem to have possession of her backpack. C. Two boys are playing a one-on-one game of basketball and are arguing over the last basket scored. D. Three boys are huddled over a handheld video game, which isn’t supposed to be on school grounds. Answer: Option B Explanation: Seeing four girls surrounding another girl, while in possession of her backpack, is the most suspicious of the incidents described.

170

Logical Reasoning

Nithra

2. Dr. Miller has a busy pediatric dentistry practice and she needs a skilled, reliable hygienist to keep things running smoothly. The last two people she hired were recommended by top dentists in the area, but they each lasted less than one month. She is now in desperate need of a hygienist who can competently handle the specific challenges of her practice. Which one of the following candidates should Dr. Miller consider most seriously? A. Marilyn has been a hygienist for fifteen years, and her current employer, who is about to retire, says she is the best in the business. The clientele she has worked with consists of some of the wealthiest and most powerful citizens in the county. B. Lindy recently graduated at the top of her class from one of the best dental hygiene programs in the state. Prior to becoming a dental hygienist, Lindy spent two years working in a day care center. C. James has worked as a dental hygienist for three years in a public health clinic. He is very interested in securing a position in a private dental office. D. Kathy is an experienced and highly recommended dental hygienist who is also finishing up a degree in early childhood education, which she hopes will get her a job as a preschool teacher. She is eager to find a job in a pediatric practice, since she has always wanted to work with children. Answer: Option B Explanation: The situation described indicates that Dr. Miller’s practice presents some specific challenges, namely that it is a busy environment with a child clientele. There is also some indication that even highly recommended, experienced hygienists might not be cut out for Dr.Miller’s office. There is nothing to suggest that Marilyn (choice a) or James (choice c) would be a good fit for Dr.Miller’s practice. Kathy (choice d) has experience and she is also interested in working with children. However, the fact that she hopes to become a preschool teacher in the not-too-distant future indicates that she might not be the kind of committed, long-term employee that Dr. Miller needs. Lindy (choice b), with her hands-on experience working with children as well as a degree from a prestigious dental hygiene program, is the most attractive candidate for the position based on the situation described. 171

Logical Reasoning

Nithra

3. Mrs. Jansen recently moved to Arizona. She wants to fill her new backyard with flowering plants. Although she is an experienced gardener, she isn’t very wellversed in what plants will do well in the Arizona climate. Also, there is a big tree in her backyard making for shady conditions and she isn’t sure what plants will thrive without much direct sunlight. Her favorite gardening catalog offers several backyard seed packages. Which one should Mrs. Jansen choose? A. The Rainbow Collection is ideal for Northeast gardens. It includes a variety of colorful perennials that thrive in cool, moist conditions. B. The Greenhouse Collection will blossom year after year if planted in brightly lit locations and watered regularly. C. The Tree house Collection will provide lush green plants with delicate colorful flowers that thrive in shady and partially shady locations. D. The Oasis Collection includes a variety of perennials that thrive in dry climates and bright sunlight. Answer: Option C Explanation: The Tree house Collection is the only package that can thrive in shady locations. Choice a requires a North-eastern climate. Choices b and d require bright sunlight. 4. Eileen is planning a special birthday dinner for her husband’s 35th birthday. She wants the evening to be memorable, but her husband is a simple man who would rather be in jeans at a baseball game than in a suit at a fancy restaurant. Which restaurant below should Eileen choose? A. Alfredo’s offers fine Italian cuisine and an elegant Tuscan decor. Patrons will feel as though they’ve spent the evening in a luxurious Italian villa. B. Pancho’s Mexican Buffet is an all-you-can eat family style smorgasbord with the best acos in town. C. The Parisian Bistro is a four-star French restaurant where guests are treated like royalty. Chef Dilbert Olay is famous for his beef bourguignon. 172

Logical Reasoning

Nithra

D. Marty’s serves delicious, hearty meals in a charming setting reminiscent of a baseball clubhouse in honor of the owner,Marty Lester, a former major league baseball all star. Answer: Option D Explanation: Since Eileen’s husband does not enjoy fancy restaurants, choices a and c can be ruled out. Choice b, although casual, doesn’t sound as though it would be the kind of special and memorable evening that Eileen is looking for. Choice d, which is owned by a former baseball star and is described as “charming” and “reminiscent of a baseball clubhouse,” sounds perfect for Eileen’s husband, who is described as a baseball fan and a man with simple tastes. 5. Mark is working with a realtor to find a location for the toy store he plans to open in his town. He is looking for a place that is either in, or not too far from, the center of town and one that would attract the right kind of foot traffic. Which of the following locations should Mark’s realtor call to his attention? A. a storefront in a new high-rise building near the train station in the center of town whose occupants are mainly young, childless professionals who use the train to commute to their offices each day B. a little shop three blocks away from the town’s main street, located across the street from an elementary school and next door to an ice cream store C. a stand-alone storefront on a quiet residential street ten blocks away from the town’s center D. a storefront in a small strip mall located on the outskirts of town that is also occupied by a pharmacy and a dry cleaner Answer: Option B

173

Logical Reasoning

Nithra

Explanation: This option is both near the centre of town and in a location (near a school and an ice cream store) where children and their parents are sure to be around. This is the only option that meets both of Mark’s requirements. 6. Rita, an accomplished pastry chef who is well known for her artistic and exquisite wedding cakes, opened a bakery one year ago and is surprised that business has been so slow. A consultant she hired to conduct market research has reported that the local population doesn’t think of her shop as one they would visit on a daily basis but rather a place they’d visit if they were celebrating a special occasion. Which of the following strategies should Rita employ to increase her daily business? A. making coupons available that entitle the coupon holder to receive a 25% discount on wedding, anniversary, or birthday cakes B. exhibiting at the next Bridal Expo and having pieces of one of her wedding cakes available for tasting C. placing a series of ads in the local newspaper that advertise the wide array of breads, muffins, and cookies offered at her shop D. moving the bakery to the other side of town Answer: Option C Explanation: This is the only option that would encourage people to think of the bakery as a shop they would visit regularly and not just on special occasions.

174

Logical Reasoning

Nithra

7. Mrs. Carson took a taxi to meet her three friends for lunch. They were waiting for her outside the restaurant when she pulled up in the car. She was so excited to see her friends that she left her tote bag in the taxi. As the taxi pulled away, she and her friends took notice of the license plate number so they would be able to identify the car when they called the taxi company. The four license plate numbers below represent what each of the four women thinks she saw.Which one is most likely the license plate number of the taxi? A. JXK 12L B. JYK 12L C. JXK 12I D. JXX 12L Answer: Option A Explanation: The four women seem to agree that the plate starts out with the letter J. Three of them agree that the plate ends with 12L. Three of them think that the second letter is X, and a different three think that the third letter is K. The plate description that has all of these common elements is a. 8. Zachary has invited his three buddies over to watch the basketball game on his wide-screen television. They are all hungry, but no one wants to leave to get food. Just as they are arguing about who should make the food run, a commercial comes on for a local pizzeria that delivers. The phone number flashes on the screen briefly and they all try to remember it. By the time Zachary grabs a pen and paper, each of them recollects a different number. Which of the numbers is most likely the telephone number of the pizzeria? A.995-9266 B.995-9336 C.995-9268 D.995-8266

175

Logical Reasoning

Nithra

Answer: Option A Explanation: All of the men agree that the first three numbers are 995. Three of them agree that the fourth number is 9. Three agree that the fifth number is 2. Three agree that the sixth number is 6; three others agree that the seventh number is also 6. Choice a is the best choice because it is made up of the numbers that most of the men agree they saw. 9. The film director wants an actress for the lead role of Lucy who perfectly fits the description that appears in the original screenplay. He is not willing to consider actresses who do not resemble the character as she is described in the screenplay, no matter how talented they are. The screenplay describes Lucy as an averagesized, forty something redhead, with deep brown eyes, very fair skin, and a brilliant smile. The casting agent has four actresses in mind. Actress #1is a stunning red-haired beauty who is 5'9" and in her mid-twenties. Her eyes are brown and she has an olive complexion. Actress #2has red hair, big brown eyes, and a fair complexion. She is in her midforties and is 5'5". Actress #3is 5'4" and of medium build. She has red hair, brown eyes, and is in her early forties. Actress #4is a blue-eyed redhead in her early thirties. She’s of very slight build and stands at 5'. Which two actresses should the casting agent send to meet the director? A. 1, 2 B. 2, 3 C. 1, 4 D. 2, 4 Answer: Option B

176

Logical Reasoning

Nithra

Explanation: Actresses #2 and #3 possess most of the required traits. They both have red hair and brown eyes, are average-sized, and are in their forties. Actress #1 is very tall and is only in her mid-twenties. She also has an olive complexion. Actress #4 is of very slight build and is in her early thirties. She also has blue eyes. 10. The neighbourhood block association has received many complaints about people knocking on doors and soliciting money for an unknown charity organization even though door-to-door solicitation is prohibited by local laws. Three residents have provided descriptions of individuals who have come to their door asking for money. Solicitor #1 is a white male, 20–25 years old, 5'9", 145 pounds, with very short brown hair. He was wearing a dark blue suit and carrying a brown leather briefcase. Solicitor #2 is a white male, 25–30 years old, 6'2", 200 pounds, with a shaved-head. He was wearing a red T-shirt and jeans. Solicitor #3 is a white male, approximately 23 years old, 5'10", slight build, with short brown hair. He was wearing a blue suit. Three days after the block association meeting, a resident noticed a man knocking on doors in the neighborhood and phoned the police to report the illegal activity. This solicitor was described as follows: Solicitor #4 is a white male, 22 years old, 140 pounds, about 5'10", with short brown hair. He was carrying a briefcase and wearing a dark suit. Based on this description, which of the three solicitations was also likely carried out by Solicitor #4? A. #1, #2, and #3 B. #1, but not #2 and #3 C. 1 and #3, but not #2 D. #1 and #2, but not #3 Answer: Option C

177

Logical Reasoning

Nithra

Explanation: The solicitor described as #2 has a shaved head and is much taller and heavier than the solicitors described as #1 and #3. Therefore, choices a and d, which include #2, can be ruled out. Solicitors #1, #3, and #4 have such similar descriptions that the correct answer is clearly choice c.

Questions for Practice 1. Zachary has invited his three buddies over to watch the basketball game on his wide-screen television. They are all hungry, but no one wants to leave to get food. Just as they are arguing about who should make the food run, a commercial comes on for a local pizze-ria that delivers. The phone number flashes on the screen briefly and they all try to remember it. By the time Zachary grabs a pen and paper, each of them recollects a different number. #1: All of the men agree that the first three numbers are 995. #2: Three of them agree that the fourth number is 9. #3: Three agree that the fifth number is 2. #4: Three agree that the sixth number is 6; three others agree that the seventh number is also 6. Which of the numbers is most likely the telephone number of the pizzeria? A. 995-9266 B. 995-9336 C. 995-9268 D. 995-8266

178

Logical Reasoning

Nithra

2. It is well known that the world urgently needs adequate distribution of food, so that everyone gets enough. Adequate distribution of medicine is just as urgent. Medical expertise and medical supplies need to be redistributed throughout the world so that people in emerging nations will have proper medical care. This paragraph best supports the statement that A. The majority of the people in the world have never been seen by a doctor B. Food production in emerging nations has slowed during the past several years. C. The medical-supply industry should step up production of its products. D. Many people who live in emerging nations are not receiving proper medical care. 3. The criminal justice system needs to change. The system could be more just if it allowed victims the opportunity to confront the person who has harmed them. Also, mediation between victims and their offenders would give the offenders a chance to apologize for the harm they have done. This paragraph best supports the statement that victims of a crime should A. Learn to forgive their offenders. B. Have the right to confront their offenders. C. Learn the art of mediation. D. Insist that their offenders be punished

Number Series Solved Problems 1. What will come in place of the question mark: 124, 228, 436, ?, 1684, 3348 Answer: 852 Explanation: The pattern is (x * 2)- 20 (124 * 2) - 20 = 248- 20 = 228 (228 * 2) - 20 = 456- 20 = 436 (436 * 2) - 20 = 872- 20 = 852 179

Logical Reasoning

Nithra

(852 * 2) - 20 = 1704- 20 = 1684 (1684 * 2) - 20 = 3368- 20 =3348 2. Find out the wrong term in the series 2, 3, 4, 4, 6, 8, 9, 12, 16 A. 16 B. 12 C. 9 D. 8 Answer : C. 9 Explanation : The given sequence is a combination of three series: i)1st, 4th, 7th terms i.e, 2, 4, 9 ii)2nd, 5th, 8th terms i.e, 3, 6, 12 iii)3rd, 6th, 9th terms i.e, 4, 8, 16 In each one of i, ii, iii, each term is twice the preceding term. So, 9 is wrong and must be replaced by (4x2) = 8 3. In the following question, various terms of an alphanumerical series are given with one or more terms missing as shown by (?).Choose the missing terms out of the given alternatives. 2Z5, 7Y7, 14X9, 23W11, 34V13, ? A. 45U15 B. 47V14 C. 47U15 D. 27U24 Answer : C. 47U15

180

Logical Reasoning

Nithra

Explanation : The first numbers increase by 5, 7, 9, 11.... The letters move one step backward. The last numbers are consecutive odd numbers. 4. Look at this series: 201, 202, 204, 207, ... What number should come next? A. 205 B. 211 C. 210 D. 208 Answer : B. 211 Explanation : In this addition series, 1 is added to the first number; 2 is added to the second number; 3 is added to the third number; 4 is added to the fourth number; and go on. 5. In this type of questions, one term in the number series is wrong. Find out the wrong term. 1, 3, 10, 21, 64, 129, 356, 777 A. 21 B. 356 C. 10 D. 129 Answer : B. 356 Explanation : The correct pattern is x 2 + 1, x 3 + 1, x 2 + 1, x 3 + 1,..... So, 356 is wrong and must be replaced by (129 x 3 + 1) i.e. 388. 181

Logical Reasoning

Nithra

6. Complete the series 95, 115.5, 138, ..., 189 A. 160.5 B. 164.5 C. 162.5 D. 166.5 Answer : C. 162.5 Explanation : Pattern in the series is, +20.5 , +22.5, Next will be +24.5 138 + 24.5 = 162.5 7. Look at this series: 8, 43, 11, 41, __, 39, 17, ... What number should fill in the blank? A. 8 B. 44 C. 43 D. 14 Answer : D. 14 Explanation : This is a simple alternating addition and subtraction series. The first series begins with 8 and adds 3; the second begins with 43 and subtracts 2.

182

Logical Reasoning

Nithra

8. In following question, a number series is given with one term missing. Choose the correct alternative and fill in the blank spaces. 29, 29, 27, 23, 25, 19, 23, 17, ?, ? A. 19, 13 B. 19, 15 C. 21, 13 D. 19, 13 Answer : C. 21, 13 Explanation : 29, 29, 27, 23, 25, 19, 23, 17, ?, ? The given series is an alternate series. The numbers in the alternate positions starting with 29 in the first position form a series of consecutive odd numbers in decreasing order. i.e. 29, 27, 25, 23, 21. The remaining numbers form a series of prime numbers in decreasing order starting with 29 i.e 29, 23, 19, 17, 13. Hence, the next two numbers in the series are 21 and 13 respectively. 9. Look at this series: What number should fill the blank? F2, ___, D8, C16, B32, A. A16 B. G4 C. E4 D. E3 Answer : C. E4

183

Logical Reasoning

Nithra

Explanation : The letters decrease by 1; the numbers are multiplied by 2. 10. In following question, a number series is given with one term missing. Choose the correct alternative and fill in the blank spaces. 0.5, 1.5, 4.5, 13.5, ? A. 45.5 B. 40.5 C. 30.5 D. 39.5 Answer : B. 40.5 Explanation : Each term of the series is obtained by multiplying the preceding term by 3. Missing number = 13.5 * 3 = 40.5

Questions for Practice 1. Look at this series: 1.5, 2.3, 3.1, 3.9, ... What number should come next? A. 4.2 B. 4.4 C. 4.7 D. 5.1 2. Look at this series: 14, 28, 20, 40, 32, 64, ... What number should come next? A. 52 B. 56 C. 96 D. 128 184

Logical Reasoning

Nithra

3. Look at this series: 2, 4, 6, 8, 10, ... What number should come next? A. 11 B. 12 C. 13 D. 14 4. Look at this series: 201, 202, 204, 207, ... What number should come next? A. 205 B. 208 C. 210 D. 211 5. Look at this series: 544, 509, 474, 439, ... What number should come next? A. 404 B. 414 C. 420 D. 445

Odd Numeral 1. Choose the number which is different from others in group. A. 29 B. 53 C. 85 D. 125 Answer: Option D Explanation: 125 is the number in the group which is a perfect cube. 185

Logical Reasoning

Nithra

2. Choose the number which is different from others in the group. A. 17 B. 27 C. 29 D. 37 Answer: Option B Explanation: Each of the numbers except 27, is a prime number. 3. Choose the number which is different from others in the group. A. 15 B. 63 C. 257 D. 195 Answer: Option C Explanation: Each of the numbers except 257, is one less than the square of a certain number. 4. Choose the number which is different from others in the group. A. 120 B. 168 C. 290 D. 380 Answer: Option D 186

Logical Reasoning

Nithra

Explanation: Each of the numbers except 380, is either one less or one more than the square of a certain number. 5. Choose the number which is different from others in the group? A. 10 + 9 B. 19 - 0 C. 95 / 5 D. 19 * 0. Answer : 19 * 0 Explanation : 10 + 9 = 19 19 - 0 = 19 95 / 5 = 19 but 19 * 0 = 0 6. Choose the odd numeral in the group? 1 (80) 9 6 (12) 2 7 (33) 4 11(112) 3 Answer : 6 (12) 2 Explanation : (i) 1 + 9 = 10 1 - 9 = -8 10 * (-8) = -80 187

Logical Reasoning

Nithra

(ii) 6 + 2 = 8 6-2=4 8 * 4 = 32 is not equal to 12. 7. Choose the number which is different from others in the group? 3749 4635 5869 7946 Answer : 3749 Explanation : In all other numbers, the sum of the first and the last digits is equal to the sum of other two digits. 8. Choose the number pair/group which is different from others? A. 18 - 45 B. 16 - 40 C. 14 - 28 D. 8 - 20 Answer: C . 14 - 28 Explanation: In all other pairs, 2nd number = 1st number * 2.5. 45 = 18 * 2. 5 = 45 40 = 16 * 2.5 = 40 28 = 14 * 2.5 = 35 (is not equal to 28) 20 = 8 * 2.5 = 20

188

Logical Reasoning

Nithra

9. Choose the number which is different from others in the group. A. 13 B. 17 C. 23 D. 27 Answer: Option C Explanation: Each of the given numbers is a prime number. But the number obtained on reversing the digits of each of the numbers except 23, is also a prime number. 10. Choose the number which is different from others in the group. A. 145 B. 399 C. 257 D. 325 Answer: Option B Explanation: Each of the numbers except 399, is one more than the square of a certain number.

Questions for Practice 1. Choose the number which is different from others in the group. A. 6 B. 12 C. 18 189

Logical Reasoning

Nithra

D. 9 E. 7 2. Choose the number which is different from the others in the group. A. 126 B. 217 C. 345 D. 513 E. 730 3. Choose the number which is different from the others in the group. A. 1532 B. 8749 C. 4268 D. 5846 E. 6137 4. Choose the number which is different from others in the group. A. 121 B. 137 C. 153 D. 177 E. 183 5. Choose the number which is different from the others in the group. A. 35 B. 49 C. 50 D. 63 E. 140

190

Logical Reasoning

Nithra

Puzzle Solved Problems 1. The Hardest Logic Puzzle Ever? If a giraffe has two eyes, a monkey has two eyes, and an elephant has two eyes, how many eyes do we have? A) 3 C) 1

B) 4 D) 2

Answer: B) 4 Explanation: 4 eyes. Here in the question, it is asked how many Eyes We have so that means here the person who has asked the question is also including the person who is suppose to give the answer. In a clear understanding, the Conversation is happening between 2 people 1st who asked the question and 2nd to whom it has been asked, which means there are 4 eyes. 2. One rabbit saw 6 elephants while going towards River. Every elephant saw 2 monkeys are going towards river. Every monkey holds one tortoise in their hands. How many animals are going towards the river? A) 14 B) 11 C) 8 D) 5 Answer: D) 5

191

Logical Reasoning

Nithra

Explanation: From the given data, 1 rabbit is going towards river not the six elephants. And these 6 elephants saw 2 monkeys are going towards river. Each monkey is holding 1 tortoise. 3. 'Rhythm of eyes' city name in India? A) Chennai C) Srinagar

B) Nainital D) Karnataka

Answer: B) Nainital Explanation: 'Rhythm of eyes' logically means Nainital in India. The other cities are: No Zip - Chennai Mr. City - Srinagar Do Acting – Karnataka. 4. 5 C in an OF? A) 5 Cards in an Old Fan B) 5 Circles in an Olympic Flag C) 5 Colors in an Old Football D) 5 Colors in an Old Flag Answer: B) 5 Circles in an Olympic Flag

192

Logical Reasoning

Nithra

Explanation: 5 C in an OF denotes 5 Circles in an Olympic Flag. These are riddles of type 12 S of the Z 52 W in a Y. 5. What is black when you buy it, red when you use it, and gray when you throw it away? A) Shoe C) Belt

B) Charcoal D) All the above

Answer: B) Charcoal Explanation: It is Charcoal which is black when we buy it or when not being used. It gets red in appearance when put in use means to say when burning. And eventually when it's been consumed by fire or after getting burned out or used up, changes into ashes which look grey in colour. 6. A man was murdered in his office. The suspects are Ericson, Maggi, Joel, Benny, Sona, Patick. A calendar found near the man has blood written 6, 4, 9, 10, 11. Who is the killer? A) Maggi C) Sona

B) Ericson D) Joel

Answer: D) Joel

193

Logical Reasoning

Nithra

Explanation: If we consider the given numbers 6, 4, 9, 10, 11 to be the month number then we can easily find the killer. 6 – June – J 4 – April – A 9 – September – S 10 – October – O 11 – November – N Hence the answer is JOEL. 7. What is 12 S of the Z? 12 S of Z represents 12 Signs of the Zodiac. It is similar to 1. 52 W in a Y

- 52 weeks in a year

2. 3 H in a C

- 3 Hands in a Clock

3. 3600 S in an H - 3600 Seconds in an Hour

4. 5 O in the W

- 5 Oceans in the World

194

Logical Reasoning

Nithra

8. They come out at night without being called and are lost in the day without being stolen. What are they? A) Light C) Stars

B) Bats D) Flights

Answer: C) Stars Explanation: Stars are what that come out at night without being called and are lost in the day without being stolen. 9. 52 C in a P? Answer : 52 Cards in a Pack Explanation: 52 C in a P represents 52 Cards in a Pack. Similar to 12 S of the Z 5 C in an OF 10. Poor people have it. Rich people need it. If you eat it you die. What is it? A) Money B) Brain C) Luxury items D) Nothing Answer: D) Nothing

195

Logical Reasoning

Nithra

Explanation: Poor people have Nothing Rich people need Nothing If you eat Nothing you die. Hence, it is Nothing.

Questions for Practice 1. What has teeth but cannot bite? A) Comb C) Zipper

B) Saw D) Gear

2. Key, Door, Lock, Room, Switch on Arrange the words given above in a meaningful sequence. A) 4, 2, 1, 5, 3 B) 1, 3, 2, 4, 5 C) 5, 1, 2, 4, 3 D) 1, 2, 3, 5, 4 3. What has a spine but no bones? A) Mirror C) Table

B) Book D) None of the above

4. If Theresa's daughter is my daughter's mother, what am I to Theresa? A) Son-in-law B) Daughter-in-law C) Grandmother D) Grand Daughter 5. ‘Brothers and sisters, I have none. But that man's father is my father's son'. How I and that man are related respectively? A) Father & Son C) Son & Father

B) Brothers D) Nephews 196

Logical Reasoning

Nithra

Ranking Test Solved Problems 1.Saran is eighteenth from the right end in a row of 50 boys.What is his position from the left end? 1. 32 2. 35 3. 33 4. 34 5. None of these Answer:3).33 Explanation : 50 – 18 = 32 Saran is 33 from the left. 2.In a class of 90 ,where girls are twice that of boys ,Shridar ranked fourteenth from the top ,if there are 10 girls ahead of Shridar ,how many boys are after him in rank? 1.23 2.26 3.25 4.22 5.None of these Answer:2.26 Explanation : No of boys = x; No of girls = 2x; x+2x = 90 => 3x = 90 x (Boys)= 30 ; 2x(Girls) = 60 197

Logical Reasoning

Nithra

Number of student behind Shridar = 90 – 14 = 76 No of girls behind Shridar = 60 – 10 = 50 No of boys behind Shridar = 76 – 50 = 26 3. Sita ranks ninetieth in a class of 68 students . What is her rank from last ? 1. 2. 3. 4. 5. None of these

50 51 49 48

Answer:1.50 Explanation: 68 - 19 = 49, Hence 50th rank from last. 4. Raji is 5 ranks ahead of Raj in a class of 46 students. If Raj’s rank is twelth from the last, what is Raji’s rank from the start? 1. 2. 3. 4. 5. None of these

29 31 28 30

Answer:4.30 Explanation: No of students ahead Raji is 30th rank from the first.

of

Raji

198

in

a rank =

46

– 17 = 29

Logical Reasoning

Nithra

5. Karthick is 6 ranks ahead of Subash who ranks sixteenth in a class of 42. What is Karthick’s rank from the last? 1. 33 2. 32 3. 31 4. 30 5. None of these Answer:1.33 Explanation: Number of Students Karthick ranks 33rd from the last.

behind

Karthick

=

42



10

=

32

6. A ranks fourth in a class. B ranks ninth from the last, If C is ninth after A and just in the middle of A and B, How many students are there in the class? 1. 33 2. 32 3. 31 4. 30 5. None of these Answer:3.31 Explanation: −−−A−−−−−−−−C−−−−−−−−B−−−−−−−− 3+1+8+1+8+1+9 = 31

199

Logical Reasoning

Nithra

7.Akil ranked seventeenth from the top and thirty seventh from the bottom in a class. How many students are there in the class? 1.53 2.45 3.54 4.52 5.None of these Answer : 1. 53 Explanation: 16+1+36 = 53 students. 8. Shakthi ranks eleventh in a class of 54 students. What is his rank from last ? 1. 2. 3. 4. 5. None of these

43 44 42 40

Answer:2.44 Explanation: 54 - 11 = 43, Hence 44th rank from last. 9. Naresh is twenty-two from the left end in a row of 47 boys. What is his position from the right end? 1. 24 2. 25 3. 23 4. 26 5. None of these

200

Logical Reasoning

Nithra

Answer:2.25 Explanation: 47 - 22 = 25, Hence 26th from the right end. 10. Reshma and Praveena are ranked ninth and thirteenth from the top in a class of 57 students. What will be theie respective ranks from the bottom of the class? 1.48,44 2.49,45 3.45,49 4.47,43 5. None of these Answer:2.49,45 Explanation: Reshma rank = 57 - 9 = 48, Reshma is 49th from the bottom. Praveena rank = 57 - 13 = 44 , Praveena is 45th from the bottom

Questions for Practice 1. Veer Das ranks 7th from the top and 28th from the bottom in a class. How many students are there in the class? A.32 B.33 C.34 D.35 E. None of these

201

Logical Reasoning

Nithra

2. During assembly the students are standing in a line. Salman Khan is 21stin order from both the ends. How many boys are there in the class? A.31 B.41 C.40 D.30 3. Priyanka Chopara ranks 17thin a class of 49 students. What is her ranks from the last? A.31 B.32 C.33 D.37 4. In a class of 35 students, Ziya is placed 7thfrom the bottom where as Sofia is placed 9thfrom the top. Shahruk is placed in between the two. What is Ziya’s position from Shahruk? A.10 B.15 C.19 D.21 5. Hritik is 7thranks ahead of Aamir Khan in a class of 39. If Aamir Khan’s rank is 17thfrom the last, what is Hritik’s rank from the start? A.10 B.12 C.15 D.16

202

Logical Reasoning

Nithra

Seating Arrangements I. Directions for Questions 1 to 5: Read the following information carefully and answer the questions given below : i. P, Q, R, S, T, U and V are sitting around a circular table facing the centre ii. R is next to the left of U and V is second to the left of R. iii. P is sitting third to the left of T. iv. Q is between S and T. 1. Which of the following is false? 1. P is fourth to the right of T. 2. U is to the immediate right of R. 3. U is third to the right of S. 4. Q is to the immediate left of S. Answer: Option 3 2. Which of the following is true? 1. R is fourth to the right of T. 2. P is to the immediate right of V. 3. S is second to the left of T. 4. Q is second to the right of V. Answer : Option 2 3. Which of the following pairs has the first person sitting to the immediate left of the second person? 1. QT 2. RP 3. VS 4. SV 203

Logical Reasoning

Nithra

Answer : Option 4 4. In which of the following options, the middle person is sitting between the other two? 1. URT 2. TUQ 3. STQ 4. None of these Answer : Option 4 5. What is the position of U? 1. Fourth to the right of S 2. To the immediate left of R 3. Between P and T 4. To the immediate right of P Answer : Option 1 II. Directions for Questions 6 to 10: Read the following information carefully and answer these questions: i. K, L, M, N, O, P and Q are sitting in a circle facing at the centre and playing cards. ii. O is neighbour of K and N. iii. Q is not between P and M. iv. P is to the immediate right of K. v. L is second to the left of Q. 6. Which of the following does not have the pair of persons sitting adjacent to each other? 1. LK 2. ML 3. NO 204

Logical Reasoning

Nithra

4. QN Answer : Option 1 7. Which of the following pairs has the second person sitting immediately to the right of the first? 1. KL 2. ML 3. OK 4. None of these Answer : Option 3 8. What is the position of P? 1. Second to the left of M 2. Second to the right of M 3. To the immediate left of K 4. To the immediate right of L Answer : Option 1 9. Who are the neighbours of L? 1. K and P 2. M and N 3. P and M 4. None of these Answer : Option 3

205

Logical Reasoning

Nithra

10 Which of the following persons are sitting adjacent to each other in clockwise order as shown? 1. LQM 2. PLM 3. MNQ 4. ONQ Answer : Option 4

Questions for Practice Eight friends A, B, C, D, E, F, G & H from different countries are sitting around a circular table facing the center. Some of these are females and others are males. Ⅰ. H, who is from USA, sits third to the right of the one from China. Three people sit between the ones from Japan and Peru. Ⅱ. B is not an immediate neighbor of the one from Japan. D sits sixth to the left of E. Only one person sits between H & B. Ⅲ. The one from Kenya is the immediate neighbor of the ones from India and China. F is sitting fifth to the left of the one from Peru, but is not an immediate neighbor of the ones from India or Kenya. Ⅳ. Two males of the group are from India and UK, and they are sitting opposite to each other. Ⅴ. E sits fifth to the left of the one from Brazil, who is one of the six females of the group. The one from Kenya is sitting opposite to the one from USA. Ⅵ. G is not an immediate neighbor of USA or China, and is sitting fourth to the left of C, the one from UK.

206

Logical Reasoning

Nithra

1. Who is sitting seventh to the right of A? A. G B. The one from Kenya C. H D. The one from Peru 2. Which country does E belong to? A. Peru B. Kenya C. USA D. India 3. If these people made to sit in the alphabetical order, vowels first and then consonants , starting from A, who will be sitting third to the left of D? A. A B. B C. C D. H 4. If A is related to UK in the same way E is related to Peru , what is related to Japan? A. A B. B C. D D. E

207

Logical Reasoning

Nithra

5. Who is sitting third to the right of the person who is fifth to the right of the one from India? A. The one from Peru B. The one from India C. D D. H

Statement and Conclusion Statement and conclusion is basically a logical reasoning section. In this section, a statement will be given followed by a set of conclusions. You need to choose the conclusion that logically follows the statement the most. Sometimes the conclusions can be directly understood by reading the statement and sometimes the reader needs to analyse it to get the indirect conclusion Solved Problems In each question below is given a statement followed by two conclusions numbered I and II. You have to assume everything in the statement to be true, then consider the two conclusions together and decide which of them logically follows beyond a reasonable doubt from the information given in the statement. Give answer: (A) If only conclusion I follows (B) If only conclusion II follows (C) If either I or II follows (D) If neither I nor II follows and (E) If both I and II follow.

208

Logical Reasoning

Nithra

1. Statements : In a one day cricket match, the total runs made by a team were 200. Out of these 160 runs were made by spinners. Conclusions: I. 80% of the team consists of spinners. II. The opening batsmen were spinners. A. Only conclusion I follows B. Only conclusion II follows C. Either I or II follows D. Neither I nor II follows E. Both I and II follow Answer : Option D Explanation: According to the statement, 80% of the total runs were made by spinners. So, I does not follow. Nothing about the opening batsmen is mentioned in the statement. So, II also does not follow. 2. Statements : The old order changed yielding place to new. Conclusions: I. Change is the law of nature. II. Discard old ideas because they are old. A. Only conclusion I follows B. Only conclusion II follows C. Either I or II follows D. Neither I nor II follows E. Both I and II follow Answer : OptionA

209

Logical Reasoning

Nithra

Explanation: Clearly, I directly follows from the given statement. Also, it is mentioned that old ideas are replaced by new ones, as thinking changes with the progressing time. So, II does not follow. 3. Statements : Government has spoiled many top ranking financial institutions by appointing bureaucrats as Directors of these institutions. Conclusions: I. Government should appoint Directors of the financial institutes taking into consideration the expertise of the person in the area of finance. II. The Director of the financial institute should have expertise commensurate with the financial work carried out by the institute. A. Only conclusion I follows B. Only conclusion II follows C. Either I or II follows D. Neither I nor II follows E. Both I and II follow Answer : OptionE Explanation: According to the statement, Government has spoiled financial institutions by appointing bureaucrats as Directors. This means that only those persons should be appointed as Directors who are experts in finance and are acquainted with the financial work of the institute. So, both I and II follow.

210

Logical Reasoning

Nithra

4. Statements:Population increase coupled with depleting resources is going to be the scenario of many developing countries in days to come. Conclusions: I. The population of developing countries will not continue to increase in future. II. It will be very difficult for the governments of developing countries to provide its people decent quality of life. A. Only conclusion I follows B. Only conclusion II follows C. Either I or II follows D. Neither I nor II follows E. Both I and II follow Answer : Option B Explanation: The fact given in I is quite contrary to the given statement. So, I does not follow. II mentions the direct implications of the state discussed in the statement. Thus, II follows. 5. Statements : Prime age school-going children in urban India have now become avid as well as more regular viewers of television, even in households without a TV. As a result there has been an alarming decline in the extent of readership of newspapers. Conclusions: I. Method of increasing the readership of newspapers should be devised. II. A team of experts should be sent to other countries to study the impact of TV. on the readership of newspapers. A. Only conclusion I follows B. Only conclusion II follows C. Either I or II follows D. Neither I nor II follows E. Both I and II follow 211

Logical Reasoning

Nithra

Answer : Option D Explanation: The statement concentrates on the increasing viewership of TV. and does not stress either on increasing the readership of newspapers or making studies regarding the same. So, neither I nor II follows. 6.Statements : In Japan, the incidence of stomach cancer is very high, while that of bowel cancer is very low. But Japanese immigrate to Hawaii, this is reversed - the rate of bowel cancer increases but the rate of stomach cancer is reduced in the next generation. All this is related to nutrition - the diets of Japanese in Hawaii are different than those in Japan. Conclusions: I. The same diet as in Hawaii should be propagated in Japan also. II. Bowel cancer is less severe than stomach cancer. A. Only conclusion I follows B. Only conclusion II follows C. Either I or II follows D. Neither I nor II follows E. Both I and II follow Answer : Option D Explanation: The statement neither propagates the diet of any of the countries nor compares the two types of cancer. So, neither I nor II follows.

212

Logical Reasoning

Nithra

7. Statements : The Government run company had asked its employees to declare their income and assets but it has been strongly resisted by employees union and no employee is going to declare his income. Conclusions: I. The employees of this company do not seem to have any additional undisclosed income besides their salary. II.The employees union wants all senior officers to declare their income first. A. Only conclusion I follows B. Only conclusion II follows C. Either I or II follows D. Neither I nor II follows E. Both I and II follow Answer : Option D Explanation: Nothing about the details of the employees' income or the cause of their refusal to declare their income and assets, can be deduced from the given statement. So, neither I nor II follows. 8. Statements : Monitoring has become an integral part in the planning of social development programmes. It is recommended that Management Information System be developed for all programmes. This is likely to give a feedback on the performance of the functionaries and the efficacy with which services are being delivered. Conclusions: I. All the social development programmes should be evaluated. II. There is a need to monitor the performance of workers. A. Only conclusion I follows B. Only conclusion II follows C. Either I or II follows D. Neither I nor II follows 213

Logical Reasoning

Nithra

E. Both I and II follow Answer : Option E Explanation: According to the statement, monitoring and evaluation of social development programmes - their function, performance and efficiency - is absolutely essential. So, both I and II follow. 9. Statements : The T.V. programmes, telecast specially for women are packed with a variety of recipes and household hints. A major portion of magazines for women also contains the items mentioned above. Conclusions: I. Women are not interested in other things. II. An average woman's primary interest lies in home and specially in the kitchen. A. Only conclusion I follows B. Only conclusion II follows C. Either I or II follows D. Neither I nor II follows E. Both I and II follow Answer : Option B Explanation: Clearly, nothing about 'other things' is mentioned in the statement. So, I does not follow, Also, since it is mentioned that programmes and magazines for women are stuffed with kitchen recipes and other household hints, it means that women have special interest in these areas. So, II follows.

214

Logical Reasoning

Nithra

10. Statements : The distance of 900 km by road between Bombay and Jafra will be reduced to 280 km by sea. This will lead to a saving of Rs. 7.92 crores per annum on fuel. Conclusions: I. Transportation by sea is cheaper than that by road. II.Fuel must be saved to the greatest extent A. Only conclusion I follows B. Only conclusion II follows C. Either I or II follows D. Neither I nor II follows E. Both I and II follow Answer : Option B Explanation: According to the statement, sea transport is cheaper than road transport in the case of route from Bombay to Jafra, not in all the cases. So, conclusion I does not follow. The statement stresses on the saving of fuel. So, conclusion II follows.

Questions for Practice 1. Statements : The manager humiliated Sachin in the presence of his colleagues. Conclusions: I. The manager did not like Sachin. II. Sachin was not popular with his colleagues. A. Only conclusion I follows B. Only conclusion II follows C. Either I or II follows D. Neither I nor II follows E. Both I and II follow

215

Logical Reasoning

Nithra

2. Statements : Women's organisations in India have welcomed the amendment of the Industrial Employment Rules 1946 to curb sexual harassment at the work place. Conclusions: I.Sexual harassment of women at work place is more prevalent in India as compared to other developed countries. II.Many organisations in India will stop recruiting women to avoid such problems. A. Only conclusion I follows B. Only conclusion II follows C. Either I or II follows D. Neither I nor II follows E. Both I and II follow 3. Statements : Nation X faced growing international opposition for its decision to explode eight nuclear weapons at its test site. Conclusions: I. The citizens of the nation favoured the decision. II.S ome powerful countries do not want other nations to become as powerful as they are. A. Only conclusion I follows B. Only conclusion II follows C. Either I or II follows D. Neither I nor II follows E. Both I and II follow 4. Statements : In a highly centralised power structure, in which even senior cabinet ministers are prepared to reduce themselves to pathetic countries or yesmen airing views that are primarily intended to anticipate or reflect the Prime Minister's own performances, there can be no place for any consensus that is quite different from real or contrived unanimity of opinion, expressed through a well orchestrated endorsement of the leader's actions. Conclusions: I. The Ministers play safe by not giving anti-government views. II. The Prime Minister does not encourage his colleagues to render their own views. 216

Logical Reasoning

Nithra

A. Only conclusion I follows B. Only conclusion II follows C. Either I or II follows D. Neither I nor II follows E. Both I and II follow 5. Statements : National Aluminium Company has moved India from a position of shortage to self-sufficiency in the metal. Conclusions: I. Previously, India had to import aluminium. II. With this speed, it can soon become a foreign exchange earner. A. Only conclusion I follows B. Only conclusion II follows C. Either I or II follows D. Neither I nor II follows E. Both I and II follow

Verbal Classification Definition All words, except one, are related to one another in some manner, i.e., three words out of the four words will be in the same classification

Solved Problems 1.Which word does NOT belong with the others? A. parsley B. basil C. dill D. mayonnaise 217

Logical Reasoning

Nithra

Answer : Option D Explanation: Parsley, basil, and dill are types of herbs. Mayonnaise is not an herb. 2.Which word does NOT belong with the others? A. inch B. ounce C. centimeter D. yard Answer : Option B Explanation: An ounce measures weight; the other choices measure length. 3. Which word does NOT belong with the others? A. tyre B. steering wheel C. engine D. car Answer: Option D Explanation: Tyre, steering wheel, and engine are all parts of a car. 4. Which word does NOT belong with the others? A. tulip B. rose C. bud 218

Logical Reasoning

Nithra

D. daisy Answer: Option C Explanation: Tulip, rose, and daisy are all types of flowers. A bud is not. 5. Which word does NOT belong with the others? A. rye B. sourdough C. pumpernickel D. loaf Answer : Option A Explanation: Loaf, sourdough, and pumpernickel are types of bread. A rye is not a bread type. Loaf - Bread that is shaped and baked in one piece and usually sliced before being eaten. Pumpernickel - Dark, dense German bread made from coarsely ground whole-grain rye. Sourdough - Leaven for making bread, consisting of fermenting dough, typically that left over from a previous batch. Rye - A wheat like cereal plant that tolerates poor soils and low temperatures. 6. Which word does NOT belong with the others? A. guitar B. flute C. violin D. cello Answer : Option B

219

Logical Reasoning

Nithra

Explanation: The guitar, violin, and cello are stringed instruments; the flute is a wind instrument. 7.Which word does NOT belong with the others? A. dodge B. flee C. duck D. avoid Answer : Option B Explanation: Dodge, duck, and avoid are all synonyms meaning evade. Flee means to run away from. 8. Which word does NOT belong with the others? A. branch B. dirt C. leaf D. root Answer : Option B Explanation: A branch, leaf, and root are all parts of a tree. The dirt underneath is not a part of the tree. 9.Which word does NOT belong with the others? A. street B. freeway C. interstate D. expressway 220

Logical Reasoning

Nithra

Answer : Option A Explanation: Freeway, interstate, and expressway are all high- speed highways; a street is for lowspeed traffic. 10.Which word does NOT belong with the others? A. heading B. body C. letter D. closing Answer : Option C Explanation: Heading, body, and closing are all parts of a letter; the letter is the whole, not a part.

Questions for Practice 1.Which word does NOT belong with the others? A. tape B. twine C. cord D. yarn 2. Which word does NOT belong with the others? A. leopard B. cougar C. elephant D. lion

221

Logical Reasoning

Nithra

3. Which word does NOT belong with the others? A. couch B. rug C. table D. chair 4. Which word does NOT belong with the others? A. couch B. rug C. table D. chair 5.Which word does NOT belong with the others? A. noun B. preposition C. punctuation D. adverb

222

Related Documents

Logical Reasoning.pdf
February 2021 1
Logical Reasoning
January 2021 1
Logical Reasoning
March 2021 0
Logical Backup.pdf
January 2021 1
Logical Reasoning
February 2021 1
Logical Reasoning
February 2021 11

More Documents from "glamroxx"